Section 4 Flashcards

1
Q

A 6-year-old has been admitted to the neurocritical care unit for a new diagnosis of posterior fossa brain tumor status post-resection. She is intubated, sedated, and paralyzed. The neurosurgeons placed an extra-ventricular drain to help with the management of elevated intracranial pressure (ICP). Her ICP monitor has been set at 20 and is draining 2-5 ml/hr. Her heart rate is 85 beats per minute, her blood pressure is 125/90 mmHg, her respiratory rate is set at 25 breaths per minute on the ventilator, and her FiO2 is 30% with an oxygen saturation of 95% as measured by pulse oximetry. Histologic evaluation reports large tumor cells with small round undifferentiated cells with mild to moderate nuclear pleomorphism and high mitotic count. What is the most common cytogenetic abnormality in this type of tumor?

A
  1. Monosomy 6
    2. Isochromosome 17q (i17q)
  2. Loss of heterozygosity of 9q
  3. A gain of chromosome X

  • The most common cytogenetic abnormality in medulloblastoma is i17q, wherein the short arm (p) is absent, and there is a gain of genetic material from the long arm (q).
  • In greater than 50% of patients, deletions in the short arm have been reported, resulting in a genotypic designation as 17pLOH, i.e., loss of heterozygosity of 17p.
  • Of note, the tumor suppressor gene, TP53, is located on chromosome 17p. However, mutations in TP53 are a low- frequency occurrence in medulloblastoma.
  • Monosomy 6 is associated with the WNT (wingless) medulloblastoma subgroup; LOH 9q is associated with the SHH (sonic hedgehog) medulloblastoma subgroup.
How well did you know this?
1
Not at all
2
3
4
5
Perfectly
2
Q

A 50-year-old woman is brought to the hospital after a witnessed seizure. She has complaints of numbness on the right side of the body. She has a history of chronic hypertension. Her current blood pressure is 130/90 mmHg. The motor examination is within normal limits. Sensory examination reveals the inability to recognize objects by touch on the right side. MRI with contrast shows an irregularly enhancing lesion of 2x2 cm in the left parietal lobe. MR spectroscopy shows increased choline. Functional MRI shows the sensory areas in the edematous brain around the tumor. Which of the following is the best treatment plan for this patient?

A
  1. Surgery and anticonvulsant medication
  2. Anticonvulsant medication, antiedema measures, and physiotherapy
    3. Surgery, antiedema measures, and sensory re-education
  3. Anticonvulsant medication and radiotherapy

  • Functional MRI (fMRI) shows the sensory areas in the edematous brain. Therefore, there is a chance of improvement with surgical removal of the tumor and antiedema measures.
  • Sensory re-education helps in improving the residual sensory deficit.
  • Planning with fMRI helps in preventing the intraoperative damage to the eloquent region, which may result in aggravation of deficits.
  • The center for stereognosis is the posterosuperior parietal cortex.
How well did you know this?
1
Not at all
2
3
4
5
Perfectly
3
Q

A 55-year-old female presents for a follow-up appointment at the spine clinic. She had an upper cervical spine injury three weeks ago and was placed into a halo brace at that time. Her vitals are taken and include blood pressure 120/80 mmHg, heart rate 80/minute, temperature 100.1 F (37.8 C), and oxygen saturation 99% on room air. Physical exam reveals some foul-smelling drainage and erythema only around the right-sided posterior pin, with no local abscess formation present; otherwise, the patient is neuro- intact throughout. The torque of each pin is checked, and all measured 8 in-lb. The patient had been receiving daily pin site care with hydrogen peroxide, up until several days ago. What is the most appropriate next step in managing this patient’s clinical presentation?

A
  1. Remove the right-sided posterior pin and suture up the pin site
  2. Keep the right-sided posterior pin in place and stress the importance for daily pin site care with hydrogen peroxide
  3. Tighten the right-sided posterior pin, and stress the importance for daily pine site care with hydrogen peroxide
    4. Prescribe an oral antibiotic, keep the right-sided posterior pin in place, and stress the importance for daily pin site care with hydrogen peroxide

  • There are no indications for the right-sided posterior pin to be removed based on the information provided. In the presence of a pin site infection, the pin should only be removed if there is an abscess present at the pin site, or if the pin is loose (with or without an abscess formation). If a pin meets the criteria for it being removed, then a new pin must be placed at a different site. Additionally, the pin site should not be sutured, especially in the setting of infection, as this would not allow for drainage and may promote abscess formation.
  • In order to get this question correct, one must first recognize this patient has a pin site infection, without an abscess formation, and that the pin is not loose. Knowing this information, the only correct answer is to leave the pin in place, and prescribe oral antibiotics along with continued daily pin site hydrogen peroxide washing.
  • If there was an abscess formation, the pin would have to be removed and replaced by another pin at a different location. If the pin was loose, it would have to be removed and replaced by another pin at a different location as well. A loose pin, in the setting of a pin site infection, should be removed and not tightened.
  • When there is a pin site infection, and the pin is not loose, it can remain in place; however, oral antibiotics must be prescribed as well. Therefore, local daily pin site care with hydrogen peroxide (although should be continued) does not completely address the pin site infection. The appropriate steps include prescribing an oral antibiotic. The right-sided posterior pin torque is at eight in- lb, which is the max and appropriate amount of torque needed. Therefore this pin does not need to be tightened, and doing so may cause iatrogenic harm. Additionally, if a pin is loose in the setting of pin site infection, it should be removed and replaced by a pin onto a different site, not tightened.
How well did you know this?
1
Not at all
2
3
4
5
Perfectly
4
Q

A 32-year-old female presents for double vision. Her symptoms started about a week ago, and she noted double vision, which is more prominent on looking to her right. Her symptoms are persistent and are now accompanied by headaches. She recovered from flu about a week ago. Her past medical history includes multiple sclerosis. Physical examination shows vital signs within normal limits. Cranial nerve examination show isocoric pupils, briskly reactive to light, and no visual field cuts. Extraocular muscle movement shows her left eye does not adduct past the midline, with horizontal nystagmus on the right eye on when looking to the right. Other cranial nerves are normal. The motor and sensory exam are normal. Which of the following is the most likely location of the lesion?

A
  1. Frontal eye fields
    2. Medial longitudinal fasciculus
  2. Superior colliculus
  3. Occipital lobe

  • In patients with internuclear ophthalmoplegia (INO) there is a disorder of conjugate lateral gaze. The affected eye shows impairment of adduction. With attempts to gaze contralaterally (relative to the affected eye), the affected eye adducts minimally, if at all. The contralateral eye abducts with nystagmus.
  • In INO, the divergence of the eyes leads to horizontal diplopia. If the right eye is affected, the patient will see double when looking to the left. Convergence is generally preserved.
  • INO is caused by a disruption in the medial longitudinal fasciculus (MLF).
  • In young patients, INO is most likely due to multiple sclerosis. In the elderly, it is more commonly caused by a brainstem stroke.
How well did you know this?
1
Not at all
2
3
4
5
Perfectly
5
Q

A 53-year-old woman presents to the clinic for evaluation. She underwent a laminectomy for chronic lower backache two years ago. Since then, she has tried multiple analgesic regimens, but her symptoms have persisted. She appears frustrated and tearful. Physical examination reveals restriction of back movements due to pain. The neurological examination is unremarkable. She takes oxycodone daily and hydromorphone- acetaminophen for breakthrough pain. Which of the following complications is most likely to occur in this patient?

A
  1. Chronic kidney disease
  2. Chronic liver disease
    3. Suicide attempt
  3. Schizophrenia

  • The clinical scenario describes a patient with chronic pain.
  • Chronic pain leads to significantly decreased quality of life, reduced productivity as well as lost wages, worsening of chronic disease, and psychiatric disorders such as depression, anxiety, and substance use disorders. Patients with chronic pain are also at a significantly increased risk for suicide and suicidal ideation.
  • Research has shown the lifetime prevalence for chronic pain patients attempting suicide to be between 5% and 14%; suicidal ideation was approximately 20%. Of the chronic patient patients who commit suicide, 53.6% died of firearm-related injuries, while 16.2% from an opioid overdose.
  • The patient’s current medications are unlikely to result in chronic liver or kidney disease. Schizophrenia is not associated with chronic pain syndrome.
How well did you know this?
1
Not at all
2
3
4
5
Perfectly
6
Q

An 80-year-old female presented with severe back pain after a ground-level fall two days ago. She states the pain is isolated in her low back and denies any radiation of pain, numbness, or tingling in her lower extremities. On examination, she is tender along the L1 spinous process. X-ray reveals the collapse of the L1 anterior vertebral height by 4mm. The patient is otherwise healthy, and the remainder of her physical exam is benign. She is given a prescription for pain medication and physical therapy. She returns seven weeks later without resolution of her symptoms, and she has been unable to perform her activities of daily living due to the pain. What is the most appropriate next step in management?

A
  1. Continued conservative therapy with the addition of opiate pain medication and rigid orthosis
  2. Biopsy of L1 to evaluate for neoplasm, followed by PET and CT scan
  3. T11-L3 posterior spinal fusion
    4. Percutaneous L1 vertebral augmentation

  • This patient is presenting with an L1 vertebral compression fracture that is causing debilitating pain despite over six weeks of conservative treatment. At this time, a vertebral augmentation procedure is indicated and should be discussed with the patient.
  • Vertebral compression fractures are the most common fragility fractures in the elderly and can cause significant physical limitations due to pain. These injuries occur primarily in osteoporotic bone and are classically associated with low energy mechanisms. They are caused by an abnormal quantity of bone, not quality of bone, leading to a decrease load to failure and propensity to fracture. The majority of these patients can be treated conservatively with multimodal pain control, physical therapy, and a gradual return to activity.
  • Vertebral augmentation is a process where cement is injected into the vertebral body at the site of the compression fracture in order to alleviate pain by improving the structural integrity and mechanical stabilization. Two described procedures for percutaneous vertebral augmentation are kyphoplasty and vertebroplasty. Of the two, the only kyphoplasty is a recommended treatment for subacute vertebral compression fractures failing conservative treatment.
  • Continuing conservative therapy is an option, but not with the addition of opiate analgesia. This could lead to dependence and adverse effects due to polypharmacy. This patient has no signs of malignancy (constitutional symptoms, weight loss), or imaging findings of lesions in the question stem. Surgical stabilization is not indicated in this patient as her spine is stable, and she is neurologically intact.
How well did you know this?
1
Not at all
2
3
4
5
Perfectly
7
Q

A 20-year-old man presents with an intense headache, nausea, and persistent vomiting that has been ongoing for two months. He is now finding it difficult to concentrate while playing his video game station. Examination reveals that he has difficulty with upward gaze and is unable to converge his pupils. He has mild papilledema. The blood work is not very remarkable. What is the most likely cause of the underlying condition?

A
  1. Acoustic neuroma
  2. Astrocytoma of the midbrain
    3. Pinealoma
  3. Aneurysm of the anterior circulation

  • A pinealoma is located near vital neural and vascular structures. The position accounts for most of its symptoms.
  • When a tumor of the pineal gland occurs, it may present with Parinaud syndrome, which is palsy of upward gaze, failure of convergence, and dissociation of light and accommodation.
  • MRI is the preferred radiological modality.
  • Other symptoms may include hydrocephalus, precocious puberty, and gait problems.
How well did you know this?
1
Not at all
2
3
4
5
Perfectly
8
Q

A 17-year-old patient had been intubated and managed in mechanical ventilation for neurogenic pulmonary edema resulting from a severe head injury. The patient has now shown good neurological improvement, with his motor score improving to 5. The chest x-ray also showed good improvement from that of the previous glass ground appearance. The treating clinician now wants to plan for the extubation of the patient. Which of the following is the best indicator for prognosticating successful extubation in the patient?

A
  1. Positive end-expiratory pressure (PEEP) 8
  2. Intracranial pressure (ICP) 20
    3. Rapid shallow breathing index (RSBI) 105
  3. Normal arterial blood gas (ABG) analysis

  • Rapid shallow breathing index (RSBI) is the ratio of the spontaneous breathing frequency (per minute) and the average tidal volume (liters) of the patient. It should ideally be taken at least three minutes after discontinuing the patient from ventilatory support.
  • RSBI of 105 breaths/min/L is taken now as one of the best predictors for prognosticating safe and successful extubation in an intubated patient.
  • A meta-analysis study has shown that the sensitivity of the RSBI in successful extubation in intubated patients was as high as 90%.
  • A motor score of five is also a reliable indicator for predicting safe extubation, and there is no need for monitoring intracranial pressure in each patient. The normal arterial blood gas analysis is only one of the indicators in the daily assessment of liberation potential.
How well did you know this?
1
Not at all
2
3
4
5
Perfectly
9
Q

A 55-year-old female patient who was previously evaluated at the pain management clinic presents for a scheduled lumbar sympatholysis using radiofrequency ablation at the right L3- L4 level. This procedure was planned due to persistent right knee pain as a result of complex regional pain syndrome following a motor vehicle collision five years ago. Past medical history is significant for coronary artery disease, hypertension, type 2 diabetes mellitus, obesity, peptic ulcer disease, and asthma. She received pre- operative clearance for the procedure from her primary care provider two weeks ago. Vital signs are stable, and she is afebrile. As the patient is prepared for the procedure and positioned prone in the fluoroscopy bed, the clinician notices an erythematous area over the right lumbar region, approximately 3x5 cm in size. It is indurated and warm to touch. When questioned, the patient reports she went swimming at a nearby lake with her family a few days ago and attributes the lumbar skin lesion to a bug bite that she has frequently been scratching. Which of the following would be the best course of action?

A
  1. Administer low-dose intravenous diphenhydramine and continue with the planned procedure
  2. Administer prophylactic antibiotics such as cefazolin 2 grams and continue with the planned procedure
  3. Continue with the planned procedure and refer the patient to follow-up with her primary care provider regarding the observed skin lesion
    4. Cancel the planned procedure and refer the patient to follow-up with her primary care provider regarding the observed skin lesion

  • The skin lesion described should raise the clinician’s suspicion for a local skin infection, especially given the patient’s medical history, which includes diabetes mellitus.
  • Infection at the procedural site is considered a strong contraindication to performing lumbar sympatholysis.
  • Relative contraindications include coagulation abnormalities, platelet dysfunction, malignancy near the treatment site, systemic infection or bacteremia, and severe cardiac and/or pulmonary disease.
  • Prophylactic antibiotics are not usually indicated for lumbar sympatholytic procedures.
How well did you know this?
1
Not at all
2
3
4
5
Perfectly
10
Q

A 65-years-old male complains of right leg pain rated 7/10 of severity. Imaging studies (x-rays and MRI) show severe L4-L5 lumbar stenosis with degenerative scoliosis and grade II spondylolisthesis. The patient has been treated with pain medication and steroid nerve root injection without positive results; he has heard about a decompression surgery alone (without fusion) and asks the clinician about his opinion. Based on the knowledge, what is the best answer for this patient?

A
  1. The decompression surgery alone is a good option for this patient as he has severe lumbar stenosis
  2. Decompression surgery with or without fusion is a good option, and both procedures have similar benefits
    3. Decompression surgery alone is a relative contraindication to this patient due to scoliosis and spondylolisthesis; therefore, the clinician recommends a decompression and fusion surgery
  3. Decompression surgery alone is the best option but has more complications than fusion surgery

  • Decompression surgery alone is a good option in severe lumbar stenosis. However, the patient has degenerative scoliosis and spondylolisthesis; both conditions are considered relative contraindications for decompression surgery without fusion.
  • Decompression surgery with fusion is the best indication for this patient as he has severe lumbar stenosis with scoliosis and spondylolisthesis.
  • Decompression surgery with fusion is the best indication to avoid iatrogenic spinal instability in this patient.
  • Decompression surgery alone is contraindicated in this case, as this procedure can increase the risk of spinal instability due to the presence of scoliosis and spondylolisthesis; therefore, a decompression followed by fusion is the best indication for this patient.
How well did you know this?
1
Not at all
2
3
4
5
Perfectly
11
Q

A 62-year-old man with a past medical history of alcohol use, 40-pack year smoking history, and diabetes mellitus is admitted to the ICU with a grade 4 subarachnoid hemorrhage (Hunt and Hess scale). Which of the following is the most appropriate screening recommendation for the patient’s siblings?

A
  1. Screening for those with hypertension
  2. Screening if one first-degree relative has a history of ruptured intracranial aneurysm and the time of presentation is 40 years of age
  3. Screening if there is a past medical history of ischemic stroke
    4. Screening if two first-degree relatives have a history of ruptured intracranial aneurysm

  • Screening for intracranial aneurysms is required if there is a family history of two or more first-degree relatives with ruptured intracranial aneurysms.
  • Screening is also required if there is a past medical history of autosomal dominant polycystic kidney disease.
  • Individuals with type IV Ehlers-Danlos syndrome, hereditary hemorrhagic telangiectasia, neurofibromatosis type 1, tuberous sclerosis, alpha1-antitrypsin deficiency, coarctation of the aorta, bicuspid aortic valve, fibromuscular dysplasia, pheochromocytoma, and Klinefelter syndrome are at a greater risk of developing intracranial aneurysms, and screening should be considered in these patients.
  • Screening for individuals with one first-degree relative with ruptured intracranial aneurysm doesn’t appear to be helpful
How well did you know this?
1
Not at all
2
3
4
5
Perfectly
12
Q

A 23-year-old man is brought to the outpatient clinic with a complaint of vague occipital headaches for the past 2 months. On examination, he is neurologically intact. A computed tomogram shows some hyperdensities in the right cerebellum. He undergoes a computed tomogram, which shows an arteriovenous malformation of size 7 x 5 cm involving the right cerebellar hemisphere and nuclei with venous drainage into the vein of Galen. Which of the following is the appropriate method of management?

A

1. Clinical observation and repeat angiogram every 5 years
2. Surgical excision as the lesion is associated with a high risk of a life-threatening bleed
3. Radiosurgery
4. Angioembolization of the nidus

  • The patient is having an arteriovenous malformation (AVM) involving the right cerebellar hemisphere.
  • Since it involves the eloquent area with a size >6 cm with deep venous drainage, the Spetzler Martin (SM) grade is V.
  • SM grade V AVMs are managed by observation and repeat angiogram every 5 years.
  • Treatment is planned only if there is progressive neurological deficits, steal-related symptoms, or associated aneurysms.
How well did you know this?
1
Not at all
2
3
4
5
Perfectly
13
Q

A 24-year old male presents to the clinic with excessive daytime sleepiness, memory problems, and trouble sleeping at night. He describes a recent episode where he was watching a funny TV show and then suddenly collapsed, going completely limp, while laughing at a joke. Which of the following structures is most likely involved in the patient’s pathology?

A
  1. Basal Ganglia
  2. Carotid Baroreceptor
    3. Pedunculopontine tegmentum
  3. Caudate nucleus

  • Cataplexy, excessive daytime sleepiness, memory problems, and sleep issues are all supportive of a diagnosis of narcolepsy.
  • Narcolepsy is thought to be related to dysfunction of the reticular activating system.
  • The pedunculopontine tegmentum is a component of the reticular activating system and contains primarily cholinergic neurons.
  • The reticular activating system is integral in human attention, arousal, and the modulation of muscle tone.
How well did you know this?
1
Not at all
2
3
4
5
Perfectly
14
Q

A 22-year-old man is brought to the emergency department after suffering a motorcycle collision. On physical examination, the patient is hemodynamically stable and has a single deep abrasion of the left leg. Surgical lavage and debridement of devitalized tissue is performed. A segmental injury of the common peroneal nerve is encountered and is repaired using a sural nerve graft from the same leg. Which of the following best identifies the maximum graft length at which any functional recovery is expected?

A
  1. 3 cm
  2. 6 cm
  3. 9 cm
  4. 10 cm

  • Nerve grafts longer than 6 cm usually provide poorer outcomes since the irrigation of the nerve decreases with its length.
  • In nerve gaps longer than 6 cm, the use of vascularized nerve grafts is recommended since they can provide better perfusion to the transferred tissue.
  • The length of the nerve gap should be considered at the maximum range of motion of the articulation that the affected nerve transverses.
  • Total recovery of the motor, sensitive, and autonomic functions of the reconstructed nerve is never achieved, although the shorter the gap, the better the outcomes.
How well did you know this?
1
Not at all
2
3
4
5
Perfectly
15
Q

A 17-year-old boy presents to the emergency department after being hit by a car. The patient is complaining of headache, double vision, and dizziness. His past medical history is insignificant except for an allergy to contrast media. On examination, vital signs are within normal limits. His pupils are isocoric and equally reactive to light. There is nystagmus more prominent when looking to the right. No facial asymmetry is noted. There are no motor or sensory deficits. A finger-to-finger and finger-to-nose test demonstrate dysmetria on the right side. The patient cannot undergo a CT angiogram as he has a severe allergy to contrast. An intracranial and neck Duplex scan is ordered. Which of the following findings would be expected from this study?

A
  1. Dissection of the internal carotid artery
  2. Dissection of the basilar artery
  3. Abnormal flow via the internal carotid artery
    4. Abnormal flow in the vertebral artery

  • The patient’s presentation is suspect for vertebral artery dissection. The Duplex ultrasound is rarely normal in a patient with vertebral artery dissection. Flow abnormalities will be seen the majority of the time.
  • Duplex ultrasound shows abnormal flow in 95% of vertebral artery dissections.
  • Dissection or signs specific to a dissection, such as segmental dilation of the vessel and eccentric channels, are seen only 20% of the time.
  • Using Duplex ultrasound, one should be able to visualize the vertebral arteries.
How well did you know this?
1
Not at all
2
3
4
5
Perfectly
16
Q

A 65-year-old man has been brought to the hospital following the sudden onset of altered sensorium, difficulty in speaking, and weakness in the right half of his body. The patient had a motor score of 5, and both the pupils were equal and reacting to light. The CT scan of the head revealed features of left-sided chronic subdural hematoma with a gross midline shift. He had a history of alcohol abuse. The treating physician decided for a burr hole evacuation of the hematoma. A single parietal region burr hole was made, and the cruciate dural incision was given, which revealed machinery oil-like hematoma egressing under high pressure. Thorough irrigation with normal saline was made until the clear fluid was visualized. The wound was closed. In the postoperative period, the patient shows sudden deterioration in his motor score to two, with anisocoria developing on his left pupil. What is the most likely etiology for this sudden adverse event occurring in the patient following the surgery?

A
  1. Seizure
  2. Rebleed
    3. Tension pneumocephalus
  3. Hypoglycemia

  • The presence of pneumocephalus is one of the frequent occurrences following the burr hole evacuation of the chronic subdural hematoma. It has been seen in up to 44% of cases in the immediate postoperative CT images following such procedures.
  • The small burr hole allows for the atmospheric air to enter into the skull secondary to the egress of the subdural hematoma, which can sometimes lead to the occurrence of tension pneumocephalus. This is more frequent among patients with a history of chronic alcohol abuse that leads to brain atrophy. The failure of brain expansion to the surface increases the odds of developing pneumocephalus.
  • The occurrence of this complication can be prevented by lowering the head end of the bed while evacuating the hematoma along with liberal saline irrigation. The placement of drain is also advocated to prevent the pneumocephalus. High flow oxygen in the postoperative period is recommended to manage most cases with pneumocephalus, except in cases with tension pneumocephalus.
  • The rebleed is rare since the hemostasis was assured while closing of the wound through copious saline irrigation. The patient did not show features of any seizurogenic activities in the patient.
How well did you know this?
1
Not at all
2
3
4
5
Perfectly
17
Q

A 50-year-old woman is being evaluated for persistent headaches. She is found to have a vascular lesion with direct drainage from arterial branches from the middle meningeal artery into a cortical vein. Which of the following possible factors best indicates surgical resection as a treatment option?

A
  1. Multiple medical comorbidities
  2. Accessibility via transarterial or transvenous approach
  3. Benign natural history based on Cognard classification system and lack of symptoms
    4. Failure of prior endovascular embolization to completely occlude the lesion

  • Surgery, endovascular embolization, and stereotactic radiosurgery are options for the intervention for dural arteriovenous fistulas. Surgery may be considered as a first-line treatment, though endovascular treatment should also be considered as first-line therapy. Failure of prior endovascular embolization to completely occlude a dural arteriovenous fistula is a reason to consider open surgical resection.
  • Observation with clinical and radiologic follow up is considered for most patients with asymptomatic and low-grade dural arteriovenous fistulas. Surgical resection is typically considered as a treatment option for a higher grade and symptomatic lesions.
  • Endovascular embolization may be performed through a transvenous approach, transarterial approach, direct access, or a combination of the above. When there is satisfactory endovascular access to the fistulous point of a dural arteriovenous fistula, endovascular embolization should be considered as a treatment option.
  • Significant medical comorbidities should be considered when deciding treatment of dural arteriovenous fistulas, as they impart higher risk for open surgical resection under general anesthesia.
How well did you know this?
1
Not at all
2
3
4
5
Perfectly
18
Q

A 54-year-old right-handed male presents with intractable headaches for the past five weeks. His workup reveals a large right temporal tumor. He is an aircraft pilot and would like surgical resection so he can resume his work as soon as possible. Which visual deficit may occur regarding surgery of a lesion in this region?

A

1. He may develop bilateral left-upper visual field deficit
2. He may develop bilateral right-upper visual field deficit
3. He may develop bilateral left-lower visual field deficit
4. He may develop bilateral right-lower visual field deficit

  • Lesions of the optic radiation in Meyer’s loop cause a bilateral upper quadrant visual field deficit of the contralateral side.
  • Visual information traveling from the inferior quadrant of the retina travel through the anterior temporal lobe under the inferior horn of the lateral ventricle.
  • Surgical resection of over 3cm of the temporal lobe will lead to a visual field deficit.
  • The quality of life is usually not affected in patients suffering from post-surgical homonymous superior quadrantanopia.
How well did you know this?
1
Not at all
2
3
4
5
Perfectly
19
Q

A 46-year-old man presents to the clinic with a complaint of pain in the neck along with a tingling sensation in the right upper forearm after a history of a fall from a height. The radiograph reveals fused vertebrae and a radiolucent line in the C6 vertebra with displacement. An initial set of labs is significant only for HLA-B27 positivity. What is the most appropriate management strategy for this patient?

A
  1. Rigid cervical orthosis
    2. Open reduction and instrumentation
  2. Halo traction
  3. Conservative management

  • This vignette reveals the diagnosis of ankylosis spondylitis spine with fractured C6 vertebrae.
  • The spine fracture in patients with ankylosing spondylitis is always a three-column fracture.
  • Three-column fractures should always be fixed and operated with posterior instrumentation.
  • Rigid orthosis is indicated in undisplaced fractures of the cervical spine in normal patients.
How well did you know this?
1
Not at all
2
3
4
5
Perfectly
20
Q

Which neurons should be stimulated by the spinal cord stimulation implant in a 55-year-old patient with bilateral lower extremity pain?

A

1. Dorsal root
2. Anterior root
3. Sympathetic nerves
4. Stellate ganglion

  • Dorsal root stimulation alleviates bilateral lower extremity pain.
  • A spinal cord stimulator supplies pulsed electrical signals to the spinal cord. It is used to control chronic pain and spinal spasticity or to augment standing and stepping capabilities. Spinal cord stimulation (SCS) can produce substantial analgesia.
  • The most common use of SCS is in failed back surgery syndrome in the United States and peripheral ischemic pain in Europe. It is indicated for the treatment of inoperable ischemic limb pain. It can modulate the sympathetic nervous system and increase norepinephrine release in refractory angina pectoris, decreasing the frequency of angina attacks. SCS is used to treat patients with frequent migraines, using electrodes implanted in the bilateral suboccipital region.
  • SCS is useful when other forms of therapy fail. It is invasive and may be associated with complications such as infection, bleeding, and dural puncture. SCS interaction with diathermy, pacemakers, MRI, and therapeutic ultrasound can result in unexpected changes in stimulation, serious patient injury or death, or device failure.
How well did you know this?
1
Not at all
2
3
4
5
Perfectly
21
Q

A 55-year-old man is brought in by emergency medical services (EMS) after being involved in a motor vehicle accident. On examination, the patient responds inconsistently and specifically to external stimuli, and he withdraws to painful stimuli. Which of the following would also be expected in this category of the Rancho Los Amigos scale (RLAS)?

A

1. Responds more to his wife
2. Responds more to the medical staff
3. Ability to follow simple commands
4. Tremor, athetosis, and involuntary movements

  • A patient with a Rancho Los Amigos score of 3 demonstrates only localized responses to external stimuli inconsistently. Voluntary motor testing will be unavailable for the therapist.
  • The patients will respond more to familiar people (friends and family) versus strangers.
  • When assessing motor control in these patients, only reflexes, rigidity, and tone can be assessed.
  • Withdrawal secondary to pain would be a sensory reflex evaluation. The patient is unlikely to be able to follow simple commands.
How well did you know this?
1
Not at all
2
3
4
5
Perfectly
22
Q

A 6-month-old infant girl was recently evaluated for increased fussiness associated with a rapidly growing head circumference. Between her four months and six-month well-child check-up, her head circumference rose from 50% to 90%. A subsequent head CT revealed enlarged ventricles with a large mass in the posterior fossa. What is the most common histologic subtype for this type of brain tumor?

A

1. Classic
2. Desmoplastic or nodular
3. Large-cell or anaplastic
4. Melanotic and medullomyoblastoma

  • In classic medulloblastoma, sheets of small round cells possessing a high nuclear-to-cytoplasmic ratio are noted. They have a high invasive tendency and occasional neuroblastic differentiation. The classic type constitutes approximately 70% of medulloblastomas.
  • Nodules of tumor cells displaying neurotic differentiation on a collagen-rich matrix characterize the desmoplastic variant. These tumors are less aggressive than the classic variant and account for 15% of medulloblastomas.
  • Large-cell anaplastic medulloblastomas, as the designation suggests, demonstrate features of anaplasia, including large tumor cells with abundant cytoplasm, pleomorphic nuclei, and prominent nucleoli. These tumors are typically located in the cerebellar vermis and are highly aggressive, demonstrating the high mitotic and apoptotic activity with large areas of necrosis. Consequently, the prognosis is especially poor with short survival times after diagnosis. They constitute approximately 10% of medulloblastomas.
  • Melanotic and medulloblastoma are rare and make up the remaining 5% of medulloblastomas.
How well did you know this?
1
Not at all
2
3
4
5
Perfectly
23
Q

A 48-year-old presented to the emergency department (ED) following a motor vehicle accident (MVA). He underwent primary and secondary surveys, which revealed tenderness over the neck region and painful neck rotations. The rest of the systemic and neurological examinations were normal. He underwent a CT scan of the head and cervical spine as part of his evaluation. CT cervical spine revealed an anterior C1 arch fracture with extension into the foramen transversarium. Which of the following is the most rational approach in the management of the patient?

A
  1. Reassurance
  2. Occipitocervical fixation
  3. C1-C2 posterior fixation
    4. CT vertebral artery angiography

  • Though the patient has a stable C1 fracture, there is an extension of the fracture into the foramen transversorium harbingering the risk of blunt vertebral artery injury.
  • Such injury has more than a 20% risk of developing ischemic strokes in the patient. So timely institution of anticoagulation therapy is of paramount importance in high-risk cohorts.
  • CT vertebral artery angiography thereby enables proper dichotomization of high-risk patients that require early anticoagulation therapy.
  • Fusion for C1 fracture is indicated only in cases of unstable features such as Jefferson subtypes or those with comminuted later mass fracture.
How well did you know this?
1
Not at all
2
3
4
5
Perfectly
24
Q

A 3-day-old neonate develops a fever, irritability, and floppy posture in the immediate postpartum period. The mother has had numerous sexual partners without barrier protection or prenatal care. The baby was born 4 hours after an amniotic sac rupture. Lumbar puncture is done, which reveals a white blood cell count of 1000 per mL with neutrophilic predominance. What is the most likely etiology?

A
  1. Escherichia coli
    2. Streptococcus agalactiae
  2. Haemophilus influenzae
  3. Listeria monocytogenes

  • The signs and symptoms in this infant are suggestive of meningitis. CSF findings are indicative of bacterial meningitis. Streptococcus agalactiae (group B streptococci) is the most common cause of neonatal meningitis in the immediate postpartum period.
  • It is most prevalent in young women with multiple partners and is most likely to infect the neonate during a protracted delivery.
  • E. coli is the second most common cause of neonatal meningitis. Its relative incidence is increasing due to screening for GBS and prophylaxis.
  • Listeria is a less frequent cause of neonatal meningitis. Haemophilus influenza and Neisseria meningitides rarely cause neonatal meningitis.
How well did you know this?
1
Not at all
2
3
4
5
Perfectly
25
Q

A 72-year-old man presents with a one-year history of lower back pain that is gradually getting worse. He now describes the pain as a six out of ten. He has a past medical history of lumbar disc herniation for which he had a discectomy one year ago, and the pain got better at first, but it never resolved completely. He also says he was prescribed several analgesics during this time with no relief. A spinal cord stimulator (SCS) is considered as a treatment option for this patient. Which of the following is a rare complication of the procedure?

A
  1. Lead migration
  2. Lead fracture
    3. Cerebro-spinal fluid leakage
  3. Seroma

  • The complication rate following spinal cord stimulator implantation is relatively high. The most common complications are due to the hardware malfunction, such as lead migration.
  • The puncture of the dura can occur during lead placement, causing a cerebrospinal fluid leak, but it is rare.
  • The puncture of the dura can occur during lead placement, with headaches occurring in up to 70 percent of cases.
  • Lead fractures can occur in above nine percent of cases being a strenuous physical activity, one of the causes.
How well did you know this?
1
Not at all
2
3
4
5
Perfectly
26
Q

A 68-year-old male with alcohol use disorder presents with mental status changes, gait abnormalities, and confusion, and he tells you he just had lunch with the president. Which ocular feature would one expect to find in this patient?

A
  1. Optic neuritis
    2. Diplopia
  2. Glaucoma
  3. Exposure keratopathy

  • The individual with Wernicke Korsakoff syndrome will usually present with ocular disturbances which may include diplopia, painless vision loss, or strabismus.
  • Gait abnormalities include a wide-based short stepped gait and an inability to stand or ambulate without assistance.
  • Mental status changes include apathy, paucity of speech, and indifference to the environment.
  • As the disorder progresses, other symptoms that may develop include agitation, anger, hallucinations, and confabulations.
How well did you know this?
1
Not at all
2
3
4
5
Perfectly
27
Q

A 76-year-old female comes to the clinic for a follow-up visit and is found to have no vision on the left eye. She is known to have a left optic nerve sheath meningioma since she was in her fifties. At that time, she had a mild visual loss and was not recommended to have surgery. At present, she has no pain, and no proptosis, the optic nerve is atrophic. Vision in the right eye is 20/30 with prescription eyeglasses. Magnetic resonance imaging showed a minimal increase in the size of the tumor. Which is the best management in this patient?

A
  1. Stereotactic radiotherapy to stop the growth
    2. Continue observation
  2. Surgical removal of the nerve and tumor to prevent spread posteriorly
  3. Chemotherapy with estrogen antagonists

  • If the eye with the optic nerve sheath meningioma (ONSM) is blind, and the tumor is confined to the orbit, the patient should be observed as they grow very slowly. Some surgeons advocate surgically resecting the tumor to avoid extension to other areas, but this patient is 76-year-old.
  • Observation is the acceptable measure in cases where the visual function is intact or remains stable, especially in patients who maintain a central visual acuity of 20/50 or better.
  • Stereotactic radiotherapy has been used as an alternative to surgery.
  • Surgical resection can be justified in cases of disfiguring proptosis where the visual function has significantly decreased or in cases of intracranial extension.
How well did you know this?
1
Not at all
2
3
4
5
Perfectly
28
Q

A 65-year-old female patient presents to the hospital with complaints of severe headaches associated with nausea and vomiting on awakening for the past few months. She also reports loss of weight and loss of appetite. She recalls having radiation therapy ten years ago for some growth, but can not precisely remember the diagnosis. On examination, she is found to have a heart rate of 100 beats/min, a blood pressure of 100/70 mmHg, respiratory rate of 20/min, and 95 % oxygen saturation on room air. Lab values, including CBC and metabolic profile, are within the normal range. CT head shows an irregular mass-like lesion, which was not present in her previous imaging four years ago. MRI revealed a 3 cm irregular mass in the left temporal lobe. Information regarding which of the following is most likely to help in establishing a diagnosis in such patients?

A

1. Previous radiation
2. Prior surgeries
3. Family history
4. Smoking and alcohol history

  • Glioblastoma multiforme is the most common aggressive primary cranial nervous system tumor. Usually more common in older men and African American race.
  • Studies have shown that a previous history of irradiation in early life is a risk factor for GBM. Although, in most cases, risk factors are not identified.
  • It is important to take a careful history, including a prior history of irradiation, to assess for risk factors.
  • No convincing evidence exists regarding associations of GBM with family history, smoking, or alcohol history.
How well did you know this?
1
Not at all
2
3
4
5
Perfectly
29
Q

A 30-year-old male golfer has had severe posterior right leg pain for the last year. He has failed conservative measures, and his right leg pain is interfering with golf and daily activities. He denies significant back pain. His priority is relief of pain while preserving his ability to play golf, if possible. MRI demonstrates a large L5/S1 disc herniation eccentric to the right side with severe foraminal stenosis, and preservation of disc space height. What is the most appropriate management strategy for this patient?

A
  1. Advise conservative treatment
    2. L5/S1 right-sided microdiscectomy
  2. L5/S1 transforaminal lumbar interbody fusion (TLIF)
  3. L5/S1 anterior Lumbar Interbody Fusion (ALIF)

  • While conservative management is generally best in young patients, this must be taken on a case by case basis. Here, the patient’s activities of daily living are severely impaired, and he has a clear, localized pathology to explain his symptoms. Hence, surgery is reasonable.
  • For cases with isolated, unilateral disc herniations without facet degeneration or instability, decompression alone without fusion is a suitable choice.
  • An anterior approach may not fully address his symptoms as his disc space height was preserved on imaging. Hence, posterior microdiscectomy is a better choice for direct pathology access.
  • Microdiscectomy does carry a small (~10%) risk of recurrence, which may need further discectomy and fusion (TLIF) in the future, but given his radicular symptoms and isolated imaging findings, microdiscectomy now is more appropriate than TLIF.
How well did you know this?
1
Not at all
2
3
4
5
Perfectly
30
Q

A patient with metastatic squamous cell carcinoma has transfacial/transnasal ethmoidectomy. Clear rhinorrhea developed postoperatively that was treated with a lumbar puncture to remove 20 mL of fluid daily for three days. The patient later suddenly became confused and lethargic. She has a motor score of 5 with a verbal response of only 2. Her CT head revealed a huge bifrontal hypodense lesion causing subfalcine herniation. What is the most important initial treatment for the patient?

A
  1. Craniotomy
  2. Observation
    3. Administration of 100 percent oxygen
  3. Needle aspiration

  • The clinical vignette is classical of a tension pneumocephalus.
  • The first step in treatment should be 100 percent oxygen.
  • Using 100 percent oxygen allows for the reduction of pneumocephalus owing to its diffusion mediated by nitrogen gradient (80% in pneumocephalus) towards blood (0% in 100% oxygen) and its subsequent disposal of the lungs.
  • Craniotomy or burr bole is justified in tension pneumocephalus with significant brain herniation encompassing a major threat to the patient’s life.
How well did you know this?
1
Not at all
2
3
4
5
Perfectly
31
Q

A 16-year-old patient is being planned for surgical excision of the intramedullary tumor of the conal region of the spinal cord. The patient is worried about the risk of losing his bowel function following the surgery. What intraoperative modality will help in safeguarding from this probable complication during the surgical resection of the tumor?

A
  1. Motor evoked potential
  2. Somatosensory evoked potential
    3. Bulbocavernosus reflex monitoring
  3. Intraoperative MRI

  • The preservation of bladder and bowel function is of paramount importance while operating on an intramedullary tumor in the conal region.
  • Intraoperative assessment of the bulbocavernosus reflex is a valid tool in safeguarding the bowel function while performing surgical excision of such a tumor.
  • Electromyography from anal sphincters following bulbocavernosus reflex helps in the intraoperative assessment of the integrity of the bowel function.
  • Motor and somatosensory evoked potentials can be used as an adjunct, but they play no role in assessing bladder bowel function. Intraoperative MRI is technically demanding and also requires tractography in safeguarding the relevant tracts.
How well did you know this?
1
Not at all
2
3
4
5
Perfectly
32
Q

A 38-year-old laborer presents with pain along the lateral aspect of his elbow. He first developed an odd ache three months ago and has now noticed that he has weakness in his hands and fingers. He denies any recent injury to the arm. He is a smoker and drinks alcohol socially. He takes acetaminophen for muscle pain. Examination reveals that he has preserved wrist extension but there is a marked loss of finger extension. Wrist flexion is within normal limits. There is no sensory loss. Nerve conduction studies are unremarkable, and an MRI did not show any mass lesion. Based on this presentation, where is the pathology is most likely located?

A
  1. Dorsum of the wrist
  2. Guyon canal
    3. Arcade of Frohse
  3. First rib

  • The posterior interosseous nerve is an extension of the radial nerve as it course on the posterior side of the forearm. The posterior interosseous nerve is motor to the wrist, finger extensors as well as the supinator muscle.
  • Sometimes the posterior interosseous nerve can become entrapped at the arcade of Frohse. The nerve often gets entrapped during repetitive supination and be associated with lateral epicondylitis.
  • The patient may present with weakness in wrist and finger extension without any sensory loss. Management includes rest and modification of hand activities.
  • If there is no improvement after 3-6 months of extensive conversative management, surgical decompression may be indicated.
How well did you know this?
1
Not at all
2
3
4
5
Perfectly
33
Q

A 65-year-old male presents to the emergency department following a high-velocity motor vehicle collision. On arrival, his Glasgow coma scale (GCS) is 6/15. Computed tomography (CT) of the head reveals diffuse cerebral edema. His cervical CT shows a fracture of the C1 vertebra with a concomitant teardrop fracture of the C6 vertebral body. Which of the following findings on magnetic resonance imaging (MRI) scan of the cervical spine can potentially have the most significant impact on the management of the patient?

A
  1. Odema over the posterior ligament complex
    2. Hyperintensity posterior to the dens in T2W and STIR sequences
  2. Hypointense marrow of the lateral mass of C1 on T2W MRI
  3. Concomitant traumatic disc with indentation of the anterior subarachnoid space only

  • The most important factor governing the management of C1 fractures is the intactness of the transverse atlantal ligament (TAL). The TAL can be assessed indirectly on plain radiographs and CT scans. However, the direct assessment of the ligament can only be made on MRI.
  • Axial CT scan can reveal bony avulsion injuries of the lateral mass. MRI cervical spine is the best modality to diagnose any damage to this vital ligament. T2WI and STIR (short tau inversion and recovery) sequences can reveal hyperintensity of the ligament at the insertional or intrasubstance level.
  • The rule of Spence is based on the assessment of the unilateral or bilateral overhang of the C1 lateral mass over the C2 superior articular process on an open mouth anteroposterior (odontoid) radiograph. If the combined overhand exceeds 6.9 mm (or 8.1 mm after correction for magnification errors), it indicates a rupture of TAL.
  • On lateral plain cervical radiographs, atlantodens interval (ADI) greater than 3 mm in adults and 5 mm in the pediatric population indicates TAL injury.
How well did you know this?
1
Not at all
2
3
4
5
Perfectly
34
Q

A 44-year-old male patient has noticed progressive weakness in both legs and difficulty in climbing steps during the last month. He recalls doing a lot of flexion and extension movements 2 months ago while helping his sister move to another apartment. He is an intravenous illegal drug user with negative HIV testing. He has numbness in the testicular area. On exam, he has weakness of plantar flexion on the left foot and dorsiflexion on the right. Cremasteric reflex is absent. Upon further questioning, he says that he is unable to have an erection in the last month. A magnetic resonance imaging is ordered and is reported with a lesion in the L2 spinal canal area with enhancement upon contrast administration. It has a dumbbell-shape appearance extending to the right paraspinal area. Which is the most probable diagnosis?

A
  1. Spinal epidural abscess
  2. Meningioma
  3. Massive herniated lumbar disc
    4. Schwannoma

  • Schwannoma commonly have a dumbbell-shape due to their propensity to grow out through the neural foramina.
  • A spinal epidural abscess usually arises as a complication of discitis or vertebral osteomyelitis. Symptoms can be acute as they can cause venous thrombophlebitis with ischemia of the spinal cord.
  • Massive herniated lumbar disc is one of the principal causes of cauda equina syndrome, but it is usually associated with the intervertebral disc space and does not exit through the foramen forming a dumbbell-shape appearance.
  • Meningioma may arise anywhere along the lumbar spinal canal and often have a dural tail.
How well did you know this?
1
Not at all
2
3
4
5
Perfectly
35
Q

A 65-year-old male presents for follow-up of neck pain radiating into his right arm that has been ongoing for the past three months. The symptoms have not responded to conservative management. An MRI of the cervical spine is obtained for further evaluation of the patient’s underlying pathology. Which of the following findings is most likely to be present on his MRI?

A
  1. Ligamentum flavum atrophy
  2. Increased cervical lordosis
    3. Osteophyte formation
  3. Increased disc height

  • The patient’s symptoms are likely caused by nerve root compression in the setting of cervical spondylosis.
  • Disc desiccation leads to a loss of disc height and can be accompanied by a disc herniation.
  • Alteration in load distribution across the cervical spine results in loss of cervical lordosis, which in turn contributes to overloading of the uncovertebral and facet joints, triggering osteophyte formation.
  • Altered load balance leads to bony and ligamentous hypertrophy and ossification.
How well did you know this?
1
Not at all
2
3
4
5
Perfectly
36
Q

A 22-year-old woman presents with severe back and bilateral leg pain. The patient is a professional gymnast. She says the pain gets worse after practice. She has no past medical history. The diagnosis of lumbosacral spondylolisthesis at L5/S1 is made, and she undergoes a laminectomy, partial correction of the deformity, and instrumented fusion. Which of the following postoperative neurological complications is most likely to occur in this patient?

A
  1. Weakness of the ankle extension
    2. Weakness of great toe extension
  2. Weakness of ankle plantarflexion
  3. Bowel or bladder dysfunction

  • Lumbosacral spondylolisthesis is seen in 4% to 6% of the population, particularly in people engaging in sports requiring back hyperextension activity (gymnasts, footballers, weight lifters). The majority of the slips occur at the L5/S1 level as the forces going through the lumbar spine are the greatest at that level, and facets are oriented in a more coronal fashion.
  • As the lumbar spine L5 nerve root exits below the L5 pedicle, it is most commonly affected by the slip. The reduction of high- grade slips is controversial; nevertheless, the same nerve root is usually affected. The weakness of the ankle extension would correlate with L4 nerve root weakness.
  • The clinical scenario suggests a higher grade of spondylolisthesis that was possibly treated with reduction and fusion. As the L5 nerve root exits below the L5 pedicle, it is at greatest risk of injury and would present with a weakness to great toe extension.
  • Ankle plantarflexion is mainly supplied by the S1 nerve root and is not the most commonly affected in cases of L5/S1 spondylolisthesis. Although bowel and bladder dysfunction may occasionally be present in patients with advanced L5/S1 spondylolisthesis, if it presents as a new finding in the postoperative period, it should be immediately investigated to exclude postoperative causes of cauda equina syndrome like a hematoma.
How well did you know this?
1
Not at all
2
3
4
5
Perfectly
37
Q

A 65-year-old male presented to the emergency department following a high-velocity motor vehicle accident. He is intubated on arrival with a GCS of E2V2M4. He has already undergone CT head, which reveals diffuse cerebral edema. His cervical CT reveals a fracture of the atlas vertebra with a concomitant teardrop fracture of the C6 vertebral body. With regard to the management of the C1 fracture, which of the following findings on the MRI scan can potentially have the greatest impact?

A
  1. Significant bony edema over the posterior ring of C1
    2. Hyperintense region just adjacent to the right lateral mass of atlas on T2WI MRI
  2. Significant hyperintense marrow on T2WI MRI within the lateral mass of C1
  3. Presence of hematoma in the anterior anterior epidural space, causing thecal sac indentation

  • The most important factor governing the management of C1 fractures is the intactness of transverse atlantal ligament (TAL). The TAL can be assessed indirectly on plain radiographs and CT scan. However, the direct assessment of the ligament can only be made on MRI.
  • Axial CT scan can reveal bony avulsion injuries of the lateral mass. MRI cervical spine is the best modality to diagnose any injury to this vital ligament. T2-WI and STIR (short tau inversion and recovery) sequences can reveal hyperintensity of the ligament at the insertional or intra-substance level.
  • The rule of Spence is based on the assessment of the uni- or bilateral overhang of the C1 lateral mass over the C2 superior articular process on open mouth anteroposterior (odontoid) radiograph. If the combined overhand exceeds 6.9 mm (or 8.1 after correction for magnification errors), it indicates a rupture of TAL.
  • On lateral plain cervical radiographs, atlanto-dens interval (ADI) greater than 3 mm in adults and 5 mm in pediatric population indicates TAL injury.
How well did you know this?
1
Not at all
2
3
4
5
Perfectly
38
Q

A 36-year-old man is a restrained driver in a high- speed motor vehicle collision. He arrives at the trauma bay alert and oriented but complaining of chest pain. He is found on cross- sectional imaging to have an aortic transection extending from the distal aortic arch to the hiatus. Which of the following interventions is most likely to improve postoperative morbidity following repair, given the extent of the injury?

A
  1. Right tube thoracostomy
  2. Preoperative lumbar puncture
    3. Preoperative spinal drain placement
  3. Simultaneous carotid-subclavian bypass

  • Spinal drain placement, often by neurosurgery, allows for decompression of the spinal cord. In this patient, an extensive stent graft is required that will cover many branches from the aorta to the anterior spinal cord.
  • This places the patient at high risk for spinal cord ischemia and resultant paralysis. The best practice to minimize this complication is spinal drain placement, allowing for decompression postoperatively.
  • During the assessment of the blunt trauma patient, bilateral chest tubes may be placed for pneumothorax or hemothorax. A left-sided effusion on an initial chest X-ray is a non-specific finding that, with a widened mediastinum, may represent an aortic transection. This should be verified with a CT angiogram.
  • Placement of the chest tube would not treat the aortic injury. Lumbar puncture would not provide a long enough benefit of spinal decompression. Although carotid-subclavian bypass may be required in about 40% of patients following coverage of the left subclavian artery during stent-graft placement, this is often a staged procedure. Additionally, this patient’s injury is distal to the great vessels. Occasionally, coverage of the left subclavian is necessary for a distal injury to obtain an adequate seal at the proximal stent graft.
How well did you know this?
1
Not at all
2
3
4
5
Perfectly
39
Q

A 43-year-old man is brought to the emergency department after a referral for the management of diabetic ketoacidosis. He has a history of type 1 diabetes, being on regular insulin since childhood. His major complaints are high-grade fever and severe headache for the past 5 days. He was prescribed amoxicillin for suspected sinusitis. His girlfriend reports that he has had decreased food intake in the past 3-4 days, and he had also skipped his regular medicines due to extreme lethargy. He also reports severe nausea and vomiting, along with mild breathing difficulty for the past 8 hours. Examination reveals an unconscious patient in a gasping state. He is immediately intubated and artificially ventilated. Further examination reveals proptosis, reddish discoloration, blackish eschar, and swelling of his right eye. His right pupil is fixed and dilated. His bedside blood glucose is 510 mg/dL, and ketones are strongly positive. The base excess is -14 mE/L. Treatment for diabetic ketoacidosis is initiated. He is hemodynamically stable and saturating 98% on room air. A T2 weighted MRI with Fat Sat sequence reveals soft tissue densities in the right sinuses and orbit, and gadolinium contrast evaluation confirms thrombosis in the right cavernous sinus. Sporangia and rhizoids were visualized with lactophenol blue staining of the tissues obtained from debridement. Which of the following is the next best step in the management of this patient?

A

1. Liposomal amphotericin B
2. Amphotericin B
3. Posaconazole
4. Voriconazole

  • This young patient in diabetic ketoacidosis with evidence of sinus/ orbit and cavernous sinus involvement merits a high consideration for rhino-orbital cerebral mucormycosis. The suspicion or diagnosis of rhino-orbital cerebral mucormycosis is a medical as well as surgical emergency since it carries high morbidity & mortality even with appropriate treatment. IV liposomal amphotericin is initiated immediately at a dose of 5 mg per Kg body weight per day, with many clinicians going up to a dose of 10 mg per Kg body weight/day. Surgical debridement is planned immediately.
  • Extensive surgical debridement is to be planned at the earliest. This will help to arrive at an early tissue diagnosis via frozen section specimens. Tissue PCR techniques also will help in reaching a diagnosis. Surgical debridement is also expected to reduce the fungal load. Extensive exploration and debridement of sinuses along with orbital decompression or exenteration is usually done. Local installation of amphotericin is often done in debrided cavities. The intracranial disease is also debrided as much as feasible. The role of surgical exploration and debridement in intracranial involvement is a bit controversial since many experts believe that mortality is inevitable. However, there are case reports of survival in early intracranial involvement.
  • Liposomal amphoterin B has better tissue penetration and minimal renal toxicities compared to parenteral amphotericin B.
  • IV posaconazole or isavuconazole is used only when the patient is not tolerating amphotericin or is not responding to amphotericin. They are used as a step-down oral continuation therapy agent, only after many weeks of IV amphotericin therapy has resulted in clinical improvement in the given patient.
How well did you know this?
1
Not at all
2
3
4
5
Perfectly
40
Q

A 3-month-old female infant is being evaluated due to progressive head enlargement. She was born after a normal delivery, although the 23-week sonogram showed a minimal cortical mantle in which the hemispheres were replaced with a fluid material. Her head has been increasing in circumference, but the fontanelle is not tense. Which of the following investigations is most likely to identify the etiology of the patient’s condition?

A

1. Digital subtraction angiography
2. Computed tomographic scan
3. Ultrasound
4. Brainstem auditory evoked response

  • The patient most likely has hydranencephaly. Most studies suggest that the damage caused to the brain is related to early internal carotid artery occlusion. As such, hydranencephaly is categorized into a group of circulatory developmental encephalopathies.
  • Digital subtraction angiography is used in some cases to evaluate the patency of the internal carotid arteries.
  • The anomaly presenting in hydranencephaly follows the anatomic distribution of the brain supplied by the internal carotid artery. Structures supplied by the posterior circulation (cerebellum, brain stem, thalamus, basal ganglia, and choroids plexus) are preserved.
  • Brainstem auditory evoked response test is used to confirm the loss of cortical function but with preserved brainstem function.
How well did you know this?
1
Not at all
2
3
4
5
Perfectly
41
Q

A 20-year-old woman with a spinal cord injury presents to the clinic for follow-up. She received olfactory stem cells in her spine as a regenerative treatment to improve her paraplegia. The therapy failed to improve her condition. Three years later, she is now found to have a mass in her spine. The pathological examination shows a cyst with respiratory epithelial lining. Which of the following is the next best step in the management of this patient?

A
  1. Exosomes derived from immune cells
    2. Exosomes derived from the same differentiated tissues
  2. Exosomes released from the mesenchymal stem cells (MSCs)
  3. Autologous genetically modified stem cells

  • Stem cells have been used for regenerative therapy which gave hope to patients of what was considered before as incurable diseases such as spinal cord injury, systemic sclerosis, Alzheimer disease, and myocardial infarction. One disadvantage of stem cell therapy is its carcinogenic potential.
  • The regenerative effect of MSCs is due to their paracrine action rather than the direct replacement of the injured tissue. So, the administration of conditioned medium or EVs derived from the stem cells has a similar regenerative capacity equivalent to that produced by the stem cells themselves.
  • MSCs derived EVs improved several conditions in experimental animals, including diabetic nephropathy and diabetic retinopathy.
  • Exosomes derived from immune cells or the same differentiated tissue have not been tried as regenerative therapy and are not expected to have a positive effect. Autologous engineered stem cells have the same oncogenic potential of the olfactory stem cells.
42
Q

A 65-year-old male comes in due to 6-month history of weakness on the right hand. He states that he has numbness on the medial aspect of the forearm, the medial aspect of the hand, the 4th digit, and the 5th digit. The provider does a physical examination, which showed atrophy of the first dorsal interosseus muscle, weakness of the adductor digiti minimi, and positive Froment sign. The left hand had no symptoms and was normal on physical examination. Which variant of the clinical entity observed in the patient causes predominant sensory deficits in similar cohorts of patients?

A
  1. Type 1
  2. Type 2
  3. Type 3
  4. Type 4

  • The patient has clinical signs and symptoms consistent with ulnar nerve neuropathy.
  • Ulnar neuropathy at the wrist and hand can range from pure sensory to pure motor deficits. Ulnar neuropathies of the wrist and hand are divided into 3 types.
  • Type III lesion is limited to the superficial branch, causing purely sensory deficits to the palmar aspect of the medial half of the fourth digit and the fifth digit.
  • Type I is a lesion of the ulnar nerve just proximal to or within the Guyon canal involving deep and superficial branches; this causes mixed motor and sensory deficits and subsequent weakness of all the ulnar hand muscles. Type II is a lesion involving the deep branch which causes a pure motor deficit with a varied pattern of weakness based on the compression site.
43
Q

A 53-year-old woman presents to the clinic for follow up. Ten years ago, she had a right parotid gland malignant tumor resection, which left her with incomplete facial paralysis of the right side. At that time she declined any further treatment. On physical examination today, she shows good eye closure but poor to no smiling or mouth angle movement. Which of the following is the most appropriate recommendation for the patient?

A
  1. Direct facial nerve repair
  2. Cable graft from sural nerve on the affected side
    3. Cross-face sural nerve graft and gracilis muscle free flap
  3. Physical therapy

  • Six months after denervation, the muscles suffer irreversible atrophy due to the loss of nerve stimulation. After this period, even if the nerve input is restored, the muscles will not be able to contract.
  • In this context, the atrophied muscles must be replaced with a functional muscle, and its innervation is secured by a sural nerve graft extension from the contralateral (unaffected) facial nerve.
  • This is because the dissection of the affected facial nerve in the presence of scarred tissue is complicated and even impossible in many cases.
  • Although physical and occupational therapy can generate some improvement, they won’t be able to reanimate the face or generate smiling or mouth angle movement.
44
Q

A 5-year-old girl is brought to the emergency department with a history of sudden onset loss of consciousness following a headache that had been present for the past 3 days. On examination, the Glasgow coma scale (GCS) score is 10/15 (E3V2M5). An emergency CT scan of the brain has been performed, which shows obstructive hydrocephalus with a mass in the pineal region. She undergoes an emergency third ventriculostomy and biopsy of the lesion. The cerebrospinal fluid turns to be positive for placental alkaline phosphatase. The histopathology shows large epithelioid cells with abundant periodic acid–Schiff (PAS) positive cytoplasm. Which of the following is the most appropriate management strategy for this patient?

A
  1. Surgical excision
    2. Radiotherapy
  2. Chemotherapy
  3. IV prednisolone only

  • The patient has a pineal region tumor with hydrocephalus.
  • The cerebrospinal fluid (CSF) positivity for placental alkaline phosphatase (PLAP) and histopathological features are suggestive of germinoma.
  • Germinomas are highly radiosensitive.
  • Resection of germinomas is difficult due to high collagen content.
45
Q

A 55-year-old woman is brought in by her son after an episode of seizures. She has a past medical history of poorly controlled hypertension and a 20 pack-year smoking history. The son denies any prior history of seizures. A CT scan of the head shows a large left frontal intracerebral hematoma and subarachnoid hemorrhage. Cerebral angiography reveals a saccular contrast-filled formation in the area of the anterior cerebral artery. The patient is intubated and sedated for airway protection. Which of the following is a risk factor for developing this condition?

A

1. Polycystic kidney disease
2. Oral contraceptive
3. Neurofibromatosis
4. Chronic beta-blocker intake

  • Patients with berry aneurysms present with the most severe manifestation of a major aneurysmal rupture, such as a subarachnoid hemorrhage to minor hemorrhage, also known as a sentinel bleed, non-hemorrhagic scenarios, or asymptomatic situations in which the aneurysm is found incidentally.
  • Seizures present in twenty-five percent of aneurysmal subarachnoid hemorrhage. They may be focal or generalized.
  • Genetic factors in the development of the berry aneurysm include connective tissue disorders that weaken artery walls, polycystic kidney disease, arteriovenous malformations, and a history of an aneurysm in two or more first-degree family members.
  • Other risk factors are untreated high blood pressure, cigarette smoking, drug abuse, including cocaine and amphetamines, which via their toxidrome, increases blood pressure extremely, heavy alcohol intake, and heavy caffeine intake.
46
Q

A man presents to the emergency department after developing intense headaches and confusion. No background information can be obtained. In the emergency department, he is found to have a Glasgow coma scale of 9. A CT brain reveals multiple hemorrhages restricted to the cortical region with no other diagnostic lesion. MRI SWI sequence revealed disseminated cortical microbleeds sparing the basal ganglion and the brainstem. What else would help establish the diagnosis?

A
  1. History of hypertension
  2. Evidence of a multi-infarct dementia
    3. Patient’s age
  3. Evidence of a cranial mass

  • Definitive diagnoses can only be made through a postmortem examination of the brain. However, “probable cerebral amyloid angiopathy (CAA)” diagnosis can be made during life utilizing imaging or tissue sampling.
  • Probable CAA (with imaging) must include multiple hemorrhages restricted to the cortical, lobar, or cortical- subcortical regions, absence of another diagnostic lesion, and age of 55 years old.
  • Gradient-echo magnetic resonance imaging (MRI) is one of the most important techniques used for diagnosis.
  • Brain biopsies are rarely utilized in the diagnosis of CAA, except when CAA-related inflammation is suspected. Hypertensive bleeds mostly involve the basal ganglia and the brainstem regions.
47
Q

A 52-year-old woman calls the clinic four days after having undergone chemical lumbar sympathectomy at the left L4-L5 level for chronic left lower extremity pain following a boating accident many years ago, resulting in the complex regional pain syndrome. She reports a tingling sensation in the groin that started the night following the procedure. She describes it as a constant irritation without any alleviating or exacerbating factors. The sensation is isolated and does not radiate down the bilateral lower extremity. She denies any motor weakness, difficulty walking, or loss of urinary or bowel control. She denies skin rash, discoloration, or discharge in the affected site. Which of the following is the most appropriate recommendation for this patient?

A
  1. Urgent visit to a gynecologist
  2. Diphenhydramine
  3. Schedule a repeat procedure
    4. Reassurance

  • Genitofemoral neuralgia is the most common complication following chemical lumbar sympathectomy with an incidence of 5-7%. It is the most common complication following lumbar sympatholysis and symptoms usually resolve on their own within a few weeks.
  • Associated symptoms of genitofemoral neuralgia are usually transient and last up to 6 weeks.
  • The genitofemoral nerve lies directly anterior to the psoas muscle.
  • Genitofermoal neuralgia can occur due to the spread of neurolytic agents on the genitofemoral nerve.
48
Q

An 80-year-old man is brought to the emergency department with a history of a hit by a brick on his head by a friend. On examination, there is a mild swelling in the left frontal region. He is conscious and oriented. His pulse rate is 86/minute, and his blood pressure is 140/90 mm Hg. He undergoes a computed tomogram scan of the head, which shows a diffuse enlargement of bilateral frontal sinuses associated with thinning of the bony walls and hyperpneumatization. What is the accurate term to describe this condition?

A
  1. Pneumocephalus
  2. Pneumorrhachis
    3. Pneumocele
  3. Pneumosinus dilatans

  • Pneumocephalus, also known as pneumatocele or intracranial aerocele, is defined as the presence of air in the epidural, subdural, or subarachnoid space within the brain parenchyma or the ventricular cavities.
  • Pneumorrhachis denotes intraspinal air.
  • Pneumocele is a focal or diffuse enlargement of any paranasal sinus, usually frontal, associated with thinning its bony walls and hyperpneumatization.
  • Pneumosinus dilatans is the same as pneumatocele, but the sinus walls are intact and normal.
49
Q

A 42-year-old man presents to the clinic with two years of worsening back pain. The pain is worse with activity and localized to the lower back. It is improved but no relieved with rest and non-steroidal anti-inflammatory medications as well as heat or ice. The pain is limiting his work as a truck loader. Imaging workup shows 35% anterolisthesis of L5 on S1 with elongation of the pars without overt fracture. Which of the following is the most accurate classification of this pathology?

A
  1. Grade II isthmic spondylolisthesis subtype A
    2. Grade II isthmic spondylolisthesis subtype B
  2. Grade II isthmic spondylolisthesis subtype C
  3. Grade II pathologic spondylolisthesis

  • Meyerding classification for spondylolisthesis is based on the percentage of slippage. Grade I is 1-25%, grade II is 26-50%, grade III is 51-75%, grade IV is 76-100%.
  • Isthmic spondylolisthesis is divided into subtype A (stress fractures of the pars), subtype B (elongation of the pars without overt fracture), and subtype C (acute fracture of the pars).
  • Pathologic spondylolisthesis is divided into subtype A (systemic causes) or subtype B (focal processes).
  • Failure of conservative management can lean one toward possible surgical stabilization of the affected spondylolisthesis.
50
Q

A 28-year-old male fell from a height of 10 feet. He has decreased strength in his legs. A full spine computed tomography scan shows the patient has loss of height of vertebral body with disruption of the anterior and posterior cortices as well as splaying of the spinous processes. What score would this patient receive on the thoracolumbar injury classification and severity score and what treatment is appropriate?

A
  1. 3, Nonoperative treatment with bracing and pain control
  2. 5, Nonoperative treatment with bracing and pain control
    3. 7, Surgical intervention
  3. 10, Surgical Intervention

  • The thoracolumbar injury classification and severity score (TLICS) is used to guide treatment and uniformity to spine fractures in the trauma setting. It evaluates fracture injury morphology, neurologic status, and posterior ligamentous complex integrity.
  • For this patient, the fracture type is burst +1, the neurologic status shows incomplete spinal cord injury +3, and posterior spinal cord injury +3, for a score of 7.
  • The TLICS treatment implications for a score of fewer than 4 points is usually nonsurgical management. A score of 4 points may be nonsurgical or surgical management. A score greater than 4 points indicates surgical management.
  • Initial management should be spine precautions, log rolling of the patient only, and application of a brace or special stabilization bed until the patient can be cleared for surgery.
51
Q

A 32-year-old male patient with a family history of autosomal dominant polycystic kidney disease (ADPKD) presents with a sudden onset headache. On arrival, the patient is holding his head complaining of excruciating pain. The patient is awake, alert, and oriented. On examination, he is afebrile, and his vital signs are blood pressure 192/114 mmHg, heart rate 112/min, and respiratory rate 18/min. A CT scan of the head reveals a hyperdense area filling the subarachnoid space around the circle of Willis. What medication should be used first in this case?

A
  1. Ketorolac
  2. Acetaminophen
    3. Nicardipine
  3. Hydrochlorothiazide

  • The headache need not be the worst of the patient’s life. Even a headache that is different from past headaches should raise one’s suspicion for subarachnoid hemorrhage. NSAIDs are generally avoided in brain bleeds due to the risk of increased bleeding.
  • Although analgesia is important to provide in this situation, blood pressure control is the most important measure while preparing for definitive treatment, possibly invasive.
  • Nicardipine and labetalol are fist line agents for blood pressure control in these patients. Systolic pressure should be kept lower than 140 mmHg.
  • Hydrochlorothiazide is a good medication for hypertensive patients not having a hypertensive emergency or SAH. HCTZ is not indicated acutely for pressure control in this pressure with SAH.
52
Q

A 65-year-old male with a history of diabetes mellitus and hypertension presents to the clinic with the complaint of difficulty in managing finances and forgetting appointments for the past one year. He feels unsteady while walking and occasionally has urinary incontinence for the past one month. On examination, he does not have any cranial nerve deficits. His mini-mental state exam score is 21/30. He has a magnetic gait. The clinical diagnosis is confirmed with magnetic resonance imaging (MRI), and he is being prepared for surgery. Which of the following factors, if present, is likely to improve his outcome after the surgery?

A

1. Aqueductal stroke volume of 45 microliters
2. Multiple white matter lesions
3. Resistance to the cerebrospinal fluid outflow of 12 mmHg/ml/min
4. The problem of forgetting things as his initial symptom

  • The patient has idiopathic normal pressure hydrocephalus (iNPH).
  • The surgical treatment of iNPH is the insertion of ventriculoperitoneal shunt.
  • Factors suggesting favorable outcomes following shunt surgery are aqueductal stroke volume greater than 42 microliters, lack of white matter lesions on magnetic resonance imaging, B-waves longer than 50% of intracranial pressure monitoring time, and resistance to cerebrospinal fluid outflow over 18 mmHg/ml/min.
  • In this scenario, the patient presented with dementia as the initial symptom, which is a poor prognostic factor following surgery.
53
Q

A 66-year-old woman presents to the clinic with gradual onset dysphagia, diplopia, and vomiting. She has a past medical history of coronary artery disease, hypertension, and diabetes mellitus. Her vital signs show a heart rate of 77/min, blood pressure of 127/70mmHg, respiratory rate of 12/min, and temperature of 98.6F. On examination, the Glasgow Coma Scale score (GCS) is 15/15. She has narrowed left pupil and nystagmus. She is referred to the hospital for admission where she develops numbness of her left face, and right arm and leg. She also develops vertigo and difficulty walking. Which of the following vessels is most likely involved in the patient’s disease?

A
  1. Posterior cerebral artery
  2. Anterior inferior cerebellar artery
    3. Posterior inferior cerebellar artery
  3. Anterior spinal artery

  • The lateral medullary syndrome is caused by a vascular event in the lateral part of the medulla oblongata. The arteries commonly involved in lateral medullary syndrome are the posterior inferior cerebellar artery or the vertebral artery. The lateral medullary syndrome is characterized by varied neurologic manifestations. Patients generally have a sudden onset, but approximately 25% may show a gradual onset.
  • The usual symptoms of lateral medullary infarction include vertigo, dizziness, nystagmus, ataxia, nausea and vomiting, dysphagia, and hiccups. Atherosclerotic disease is the leading cause. Other causes include cardiac embolism, arterial dissection, small vessel disease, etc.
  • Patients have impairment of pain and thermal sensation over the contralateral side of the trunk and limbs, impairment of pain and thermal sensation over the ipsilateral face, ipsilateral Horner syndrome, and ipsilateral limb ataxia.
  • Anterior inferior cerebellar artery pathology can lead to lateral pontine syndrome. Anterior spinal artery infarct causes bilateral pain and temperature loss and quadriparesis. Posterior cerebral artery infarct can lead to diplopia, visual field defects, dysphagia, vertigo, alteration in consciousness, memory impairment, or difficulty reading.
54
Q

A 28-year-old female presents to the clinic complaining of shooting pain that goes from her left buttock to her posterior left thigh. The patient stated that she has difficulty sitting for a prolonged amount of time. She had an MRI of the lumbar spine done recently, which was negative for any pathologies. On physical exam, she is very tender to palpation on her left gluteal muscles without tenderness anywhere else. What is additional physical exam maneuver best used to aid in the diagnosis of this patient’s condition?

A
  1. FABERE test
    2. FAIR test
  2. Straight leg raise test
  3. Femoral nerve stretch test

  • The FAIR (flexion, adduction, and internal rotation) test can reproduce symptoms of piriformis syndrome.
  • This involves placing the patient in the supine position and having them flex their hip and bring it across the midline and internally rotating the lower extremity, which will place tension on the piriformis muscle.
  • The one major difference between sciatica and piriformis syndrome is that the latter rarely presents with neurological deficits.
  • On the physical, localized tenderness around the buttock area is very common.
55
Q

A 65-year-old man comes to the office due to dizziness, abnormal gait, and double vision for the past few days. His previous history is positive for type 2 diabetes mellitus and hypertension, for which he is on regular medication. Family history is positive for ischemic heart disease in his father. His pulse is 67 beats/min, blood pressure is 128/76 mmHg. On physical examination of the visual system, his right eye abducts and exhibits horizontal nystagmus, but his left eye doesn’t move. When asked to look left, both eyes appear normal. Convergence appears normal in both eyes. Which of the following is the most likely anatomical site for the characteristic neurological entity observed in the patient?

A
  1. Edinger-Westphal nucleus
  2. Oculomotor nerve
  3. Trochlear nerve
    4. Medial longitudinal fasiculus

  • The patient has signs and symptoms characteristics of internuclear ophthalmoplegia (INO).
  • Medial longitudinal fasciculus allows for cross-communication between CN III and CN VI. It is extensively myelinated and coordinated both eyes to move in the correct direction.
  • The most common causes of INO are multiple sclerosis and ischemic stroke. Her risk factors of type 2 diabetes mellitus, hypertension, and family history of cardiovascular disease make the diagnosis of brainstem infarct highly likely.
  • Edinger-Westphal (EW) nucleus is the parasympathetic efferent for the eye. A lesion in the EW nucleus results in an ipsilateral fixed and dilated pupil. Cranial nerve III lesions would result in a “down-and-out” gaze, ptosis, and absent light reflex. Trochlear nerve lesion would result in a palsy of the superior oblique muscle of the eye.
56
Q

A 45-year-old male, who is a head loading worker by profession and a smoker, presents to the outpatient department with the complaint of sudden onset neck pain after lifting a heavy load on his head 3 days ago. The pain has continued since then and is radiating along the right arm and forearm. On examination, he has reduced power in the right biceps (4/5) and biceps jerk is diminished. He needs a Spurling test for further evaluation. Which of the following represents the correct technique as described in the current literature and the corresponding positive test finding?

A
  1. Passive cervical flexion and rotation with axial compression that results in the reproduction of radiculopathy symptoms in the upper extremity contralateral to the direction of head rotation.
  2. Passive flexion of the head resulting in an electric shock-like sensation down the spine
  3. Passive lateral flexion of the head with axial compression resulting in localized pain in the cervical spine
    4. Passive cervical extension with rotation to the affected side and axial compression that results in the reproduction of radiculopathy symptoms in the upper extremity ipsilateral to the direction of head rotation

  • In current literature, the Spurling test is described as a passive cervical extension with rotation to the affected side and axial compression that results in the reproduction of radiculopathy symptoms in the upper extremity ipsilateral to the direction of head rotation.
  • The Spurling test was originally defined as passive lateral flexion and compression of the head, though this is no longer recognized as the proper technique.
  • Passive cervical flexion resulting in electric shock-like paresthesia is Lhermitte sign and may indicate multiple sclerosis plaques or other intramedullary pathology.
  • The Spurling test is designed to exaggerate cervical nerve root compression to recreate radiculopathy symptoms, and a positive result should prompt further testing and imaging.
57
Q

A patient was being managed for posttraumatic hydrocephalus with the placement of external ventricular drainage (EVD). Since the hydrocephalus persisted despite two attempts of EVD weaning, a plan was made for placing a ventricular peritoneal (VP) shunt. What would be the ideal point for the placement of the proximal shunt tube following ventriculostomy to minimize its malfunction due to its obstruction?

A
  1. Keen’s point
  2. Dandy’s point
    3. Kocher’s point
  3. Frazier’s point

  • The placement of the ventriculostomy point at the Kocher’s point has been shown to decrease the risk of its subsequent obstruction by at least 33%.
  • The chandelier pattern of the proximal shunt prevents its contact with the choroid plexus, thereby minimizing its obstruction following the risk of it being ensheathed by the same.
  • This phenomenon has been shown to increase the lifespan of the shunt while compared to those that have been placed from the posterior points like Keen, Dandy, and the Frazier points.
  • In posteriorly placed proximal shunts, the tip seems to lay on the floor of the ventricles close to the choroid plexus, thereby increasing the odds of its subsequent occlusion.
58
Q

A 17-year-old male presents to the clinic with a history of numbness and weakness of his right hand and wrist muscles. Three days ago, he was out partying and passed out on a chair in a friend’s house. The next morning he woke up with weakness of his right wrist and fingers of the right hand. On physical exam, he showed weak triceps (power 2/5) and weak wrist extension (power 2/5) on the right side with no weakness on the left. He had impaired light touch and pinprick sensations on the dorsal forearm extending to the dorsum of the right hand and the right first three digits. The clinician suspects a right radial nerve injury. Electrophysiological testing done at three weeks reveals preserved nerve conduction distal to the injury site and no evidence of fibrillations potential. What will the clinician tell the patient about his injury prognosis?

A
  1. His injury initiated a process called Wallerian degeneration and may take 1 year for recovery
  2. His radial nerve was transected and will require surgical anastomosis
    3. He had a transient episode of neurological dysfunction that is expected to improve within weeks
  3. His nerve is expected to regenerate at a rate of 0.1 mm/day

  • Preserved nerve conduction distal to the injury site, and no evidence of fibrillations potential indicate that the injury is neurapraxia.
  • Neurapraxia is a transient conduction block with no disruption of external or internal nerve continuity.
  • In neurapraxia, there is no Wallerian degeneration (which is seen in axonotmesis or neurotmesis). Regeneration of the nerve is not required for recovery.
  • The loss of function is temporary and tends to recover within weeks to months at most. No surgical intervention is typically required.
59
Q

A 56-year-old male is undergoing a right parotid tumor biopsy under local anesthesia. During the procedure, it is noticed that an iatrogenic transection of a nerve coursing through the gland has occurred. Which is the most significant deficit expected from this injury?

A
  1. Pain on taking the first bite of food
  2. Loss of sensation to the midface
    3. Inability to fully close the eye
  3. Oral incompetence

  • The patient had an iatrogenic transection of the facial nerve as it courses through the parotid gland as she can not close the eye.
  • Direct end-to-end coaptation offers the best chance of functional recovery in a facial nerve injury.
  • The facial nerve innervates the orbicularis oculi muscle. It also innervated the muscles involved in facial expression.
  • The motor function of the trigeminal nerve is for the muscles of mastication, the tensor tympani, mylohyoid, tensor veli palatini, and the anterior belly of the digastric.
60
Q

A 27-year-old man presents with nasal deformity and bleeding after being involved in a fight in a pub. On examination, the patient is alert and orientated to time, place, and person. There are tenderness and crepitus of his nose bones and nasal base widening. Which of the following clinical findings would require cerebrospinal fluid (CSF) examination and a neurology consultation?

A
  1. Crepitus
  2. Epistaxis
    3. Rhinorrhea
  3. Nasal shortening

  • Nasal fractures are the most frequent facial fracture in adults with midface blunt trauma as the most common mechanism. It is usually the result of road traffic accidents, sports injuries, falls, and physical fights.
  • The presence of rhinorrhea may raise the suspicion of cerebrospinal fluid (CSF) leak, and thus the cerebral fluid must be examined, and neurology consulted.
  • Typical physical findings in nasal fractures include tenderness, crepitus, nasal shortening or depression, step-offs, and nasal base widening.
  • Diagnosis is usually made upon clinical evaluation, by history taking and physical examination, while imaging is rarely needed.
61
Q

A 71-year-old man presents to the office with a history of severe stenosis of multiple vertebrae of the C5 and C7 vertebrae. He complains of neck stiffness, and numbness and tingling down his left arm. His pain has gradually been worsening over time. He denies any decreased sensation or weakness in his right arm. He recently finished physical therapy for the pain, and it reduced his pain somewhat, but he still has difficulty sleeping due to neck pain. Oxycodone and gabapentin have provided minimal relief as well. On physical exam, his strength, sensation, and deep tendon reflexes are all within normal limits for his upper extremity. What is the most common transient side effect of the next best step in the management of this patient?

A
  1. Paralysis
    2. Worsening neck pain
  2. Vision changes
  3. Upper extremity weakness

  • A transient increase in radicular pain was seen in up to 18% of patients following injection.
  • The possible mechanism of the transient increase in pain may be the epidural needle coming in contact with associated nerve roots and spinal nerves.
  • This complication is more commonly reported with a transforaminal cervical epidural steroid injections.
  • The four most common side effects following a cervical epidural are neck pain (6.7%) headache (1.7%), insomnia (1.7%), and vasovagal reaction (1.5%).
62
Q

A 45-year-old woman presents with severe weakness of her right upper limb. On examination, there is severe weakness in shoulder abduction and elbow extension (0/5) with normal strength in elbow flexion with the forearm in supination. Based on these findings which of the following is suspected?

A
  1. C6 cervical radiculopathy
  2. Radial neuropathy
  3. Upper trunk brachial plexopathy
    4. Posterior cord brachial plexopathy

  • Weakness of shoulder abduction (deltoid, C5/C6, upper trunk, posterior cord) and elbow extension (triceps, C6/7/8, upper and mid trunk, posterior cord) fits best with a posterior cord injury as elbow flexion (primarily controlled by the biceps brachii and brachialis, C5/6, upper trunk, lateral cord) is intact arguing against a C5 or C6 radiculopathy or an upper trunk brachial plexopathy. A radial neuropathy would not be expected to cause weakness in shoulder abduction.
  • Acute radial neuropathies at the spiral groove are sometimes difficult to distinguish from a central nervous system lesion producing arm weakness. Most central disorders that cause arm weakness tend to affect the extensors greater than the flexors. Thus, weakness in extension of the wrist (ie, wrist drop) and fingers may be an ambiguous finding. Relative preservation of triceps strength and sensory loss isolated to the dorsum of the hand are helpful findings indicative of radial neuropathy at the spiral groove. Most importantly, the brachioradialis should also be weak in a radial nerve lesion, but is typically less affected in a central disorder.
  • Conservative treatment is generally the rule for patients with a one-time compression injury to the radial nerve (such as a radial Saturday night palsy). Physical therapy, wrist splinting (to maintain function), and pain management are most important. Prognosis for full recovery is generally good.
  • In patients with severe injury secondary to trauma, follow-up examinations and electromyography (EMG) may be warranted. Severe ongoing nerve compression or transection may be difficult to distinguish from a single episode of injury that occurred at the time of trauma. Lack of any improvement, either clinically or electrophysiologically, warrants further imaging or surgical exploration.
63
Q

A 45-year-old man presents to a pain clinic with features of complex regional pain syndrome. The pain, despite pain medications, has disturbed his quality of life. Owing to the refractory nature of his pain, the treating clinician schedules him for an ultrasound-guided stellate ganglion block. The patient is very hopeful about getting rid of his pain following the procedure. What is the best clinical indicator to help predict a complete sympathetic block to assure a positive outcome in the patient?

A
  1. Presence of Horner syndrome
    2. Ipsilateral warming of more than 2 C
  2. Ipsilateral flushing and lacrimation
  3. Drop in the temperature of the ipsilateral tympanic membrane

  • The blockage of T2 sympathetic ganglion is imperative in assuring complete sympathetic denervation of the upper limb. Ipsilateral warming more than 2 C was observed in more than 70% of patients following the complete sympathetic blockade.
  • The ipsilateral warming of more than 2 C is taken as the definitive sign of complete sympathetic denervation in the upper arm. The studies have shown the temperature varaibility between one and a half to seven and a half degrees Celcius.
  • The greater rise in temperature is seen in the late stages of complex regional pain syndrome owing to the concurrent presence of vasoconstriction in the patients. This temperature variability also increases with the number of procedures within the patient.
  • Horner syndrome only helps in confirming the presence of the stellate ganglion. The horner syndrome can also occur without the sympathetic blockage, even if only the upper half of the ganglion is blocked. The flushing and lacrimation are only additional and no conclusive signs to confirm a sympathetic block. Drop in the temperature of the ipsilateral tympanic membrane is also a reliable sign, but the increase in the temperature within 10 minutes of the procedure is the limiting factor.
64
Q

A 36-year-old man was a restrained passenger in a motor vehicle collision. He has significant bruising across his chest and abdomen, and extreme pain at his thoracolumbar junction on palpation. The provider on radiological investigation notes splaying of the spinous processes at the level of the injury, but no fracture at the vertebral bodies. What is the mechanism of this injury?

A
  1. Axial compression type injury
    2. Flexion/distraction type injury
  2. Rotational type injury
  3. Translational type injury

  • A chance fracture of the spine is a flexion/distraction injury of the spine that can result in a bony or ligamentous injury centered at the abdominal viscera.
  • In a chance fracture of the spine, the anterior column fails under compression, and the middle and posterior columns fail under tension.
  • Ligamentous bony chance fractures are more difficult to heal and require surgical decompression and stabilization. This is most appropriately managed from a posterior approach.
  • Bony chance fractures are more stable than ligamentous chance fractures and may be treated non-operatively. Despite this, providers should still follow closely for evidence of nonunion or kyphosis.
65
Q

A 23-year-old medical student presents to the hospital with a sudden onset headache associated with gait instability. The patient can recount that the headache started about 30 minutes ago while she was eating lunch, and her unstable gait seemed to be apparent only after this. Apart from an appendectomy at age 16, she has no significant past medical or surgical history, and she takes the oral contraceptive pill regularly. On examination, she is fully conscious, and her vitals are stable. She has no change in tone or power in all four limbs, though she does have an ataxic gait. A CT head is performed and reveals a hyperdense lesion within the cerebellum with areas of increased attenuation within the lesion. There is some surrounding hypodensity, but ventricles do not appear enlarged, and there is no intraventricular blood. What is the most appropriate line of action in the care of this patient?

A

1. Admit the patient, contact the neurosurgical and neurology team, arrange for admission blood tests, and arrange vascular imaging, i.e., a CT angiogram +/- MRI/MRA
2. Admit the patient, contact the neurosurgical team, and prep the patient for urgent posterior fossa craniectomy
3. Arrange for the patient to have blood tests and a lumbar puncture. If this is negative for xanthochromia, this patient can be discharged with analgesia
4. Admit the patient, arrange for admission blood tests, inform the neurosurgical team, start treatment for a subarachnoid hemorrhage

  • A young patient with intracerebellar hemorrhage (ICH) and no bleeding diathesis should always raise suspicion of an underlying lesion such as a vascular abnormality. The increased attenuation within the clot on the CT head may be areas of calcification within vessels and increases the likelihood of a vascular malformation being the cause of the ICH. This patient is currently stable and does not need to be rushed to the theater for surgery. In stable patients, it is important to complete investigations that may assist with their further care.
  • Posterior fossa arteriovenous malformations (AVMs) are rare and account for 7 to 15% of all intracranial AVMs. AVMs within the cerebellum make up the majority of posterior fossa AVMs.
  • An appropriate investigation of a possible vascular malformation includes an MRI/MRA, CT angiogram, and a cerebral catheter angiogram or digital subtraction angiogram (DSA). The latter studies are more invasive. However, their dynamic nature allows for the investigation of vessel filling at different times and helps identify feeding and drainage vessels, which is very useful in surgical planning.
  • Although this patient is currently stable and does not have an altered conscious level, she has just had a significant ICH that requires admission for frequent neuro-observations as the ICH could progress and the patient could deteriorate. The patient also requires further investigations for the cause of her ICH, and these should initially be done as an inpatient. Though she has a history of a sudden-onset headache, her CT head revealed the cause of this and did not show signs of subarachnoid blood.
66
Q

A 24-year-old female completes the Boston Marathon in 4 hours and complains of a progressive headache a few hours later. She became increasingly lethargic and was brought to the emergency department. The patient has no past medical history except for oral contraceptives. Computed tomography (CT) is remarkable for an empty delta sign. Which of the following is the appropriate next test?

A
  1. Transesophageal echo
  2. Lower extremity ultrasound
    3. Factor V Leiden mutation
  3. Ultrasound of the carotids

  • An empty delta sign is seen in cerebral venous thrombosis.
  • Risk factors for this include dehydration with resultant decreased intravascular volume, malnutrition, sepsis, and thrombophilias.
  • Cerebral venous thrombosis can be caused by Factor V Leiden mutation, antithrombin III deficiency, hyperhomocysteinemia, prothrombin gene mutation, and deficiencies of proteins S and C.
  • A young, active marathon runner is unlikely to have carotid vessel disease or a lower extremity deep venous thrombosis. A transesophageal echocardiogram would be useful in working up a patient with an ischemic stroke or if there is a concern in the setting of sepsis for endocarditis.
67
Q

A 70-year-old man presents with worsening lower back pain over the last two years. He says that walking long distances triggers his pain and finds comfort when sitting down or leaning over a trolley during his supermarket shop. He has a past medical history of hypertension and hyperlipidemia, well-controlled on medication. Which of the following spinal disc levels is most likely implicated?

A
  1. T12/L1
  2. L3/L4
  3. L5/S1
    4. L4/L5

  • The L4/L5 disc level is the most common spinal segment to be affected by degenerative spondylolisthesis.
  • This is where the L4 vertebral body has a true displacement forward in relation to the L5 vertebral body, referred to as an anterolisthesis. Posterior displacement or retrolisthesis is also possible but is much less common.
  • Degenerative spondylolisthesis occurs as part of the aging process, resulting in disc degeneration and arthritic changes that cause the anterior slippage.
  • This is in contrast to other forms of spondylolisthesis, such as congenital, traumatic, or isthmic, where there is typically disruption of the posterior elements such as the pars interarticularis or pedicles.
68
Q

A 17-year-old man is brought to the emergency department following a fall from a bike one hour ago. On examination, he is fully conscious and oriented. He has a bluish discoloration and swelling in the forehead. A computed tomogram scan of the brain shows a fracture of the anterior and posterior walls of the right frontal sinus. There is no intracranial bleed. The next day he starts having clear watery fluid coming out of the right nostril. A repeat CT scan shows a hypodense collection adjacent to the fracture site. What is the likely Hounsfield unit of that collection?

A

1. -1000
2. -100
3. 0
4. +1000

  • The patient has a fracture involving both tables of the frontal sinus.
  • The next day he developed cerebrospinal fluid (CSF) rhinorrhea.
  • CSF rhinorrhea can cause pneumocephalus formation.
  • The Hounsfield unit of air is -1000.
69
Q

A 38-year-old woman patient presents with seizures. She has been complaining of progressive headaches for the past month, accompanied by nausea. For the past two days, she was drowsy and has decreased appetite. She denies any fever or infection. She has a past medical history of breast cancer diagnosed two years ago and had a radical mastectomy, chemotherapy, and radiation therapy. Vital signs show oxygen saturation 98% on room air, respiratory rate 18 per minute, heart rate 96 beats per minute, blood pressure 140/90 mmHg, and temperature 98.5 F. On examination, she opens her eyes to pain, has an incomprehensible verbal response, and localizes to pain. There are intermittent episodes of preferential gaze to the right with tonic-clonic movements of the right upper extremity with secondary generalization. Babinski sign is positive bilaterally. A head CT is normal. Which of the following are the most likely findings on the cerebrospinal fluid (CSF) examination?

A

1. Elevated opening pressure, WBC 3 cells/mm3, 100% lymphocytes, glucose 55 mg/dL, protein 150 mg/dL
2. Elevated opening pressure, WBC 2500 cells/mm3, 90% neutrophils, glucose 23 mg/dL, protein 250 mg/dL
3. Normal opening pressure, WBC 68 cells/mm3, 100% lymphocytes, glucose 55 mg/dL, protein 65 mg/dL
4. Normal opening pressure, WBC 5 cells/mm3, 100% lymphocytes, glucose 65 mg/dL, protein 40 mg/dL

  • Malignancy from the breast, lung, melanoma, or even a primary brain cancer may cause carcinomatous meningitis. The survival rate is only a few weeks at best.
  • The most common manifestations are headaches and seizures.
  • Lumbar puncture is the standard diagnostic procedure. Common findings include elevated opening pressure and elevated protein levels. Malignant cells are seen in 60% to 70% of cases after three attempts.
  • The contrast-enhanced MRI is the imaging of choice as a CT scan may not be sensitive enough to pick up leptomeningeal enhancement.
70
Q

A 7-year-old boy is brought to the hospital by his father for the difficulty in standing. The father says that this is his son’s first visit to the pediatrician. He states that his son has difficulty standing up from a seated position and prefers crawling over walking. He explains that his son is adopted and that he has no information about his real parents. The patient’s past medical history and vaccination status are also unknown. Examination shows vitals within normal limits. Bilateral hypertrophied calves are observed. The assessment for power reveals decreased power in all muscle groups. What is the next best step in the diagnosis of this patient?

A
  1. X-ray of the lower limbs
  2. Work-up for metabolic diseases
  3. Muscle biopsy
    4. Genetic testing

  • Pseudohypertrophy of the calves and suggestive Gowers sign leads to the diagnosis of Duchenne muscular dystrophy.
  • Duchenne muscular dystrophy is an X-linked recessive disorder associated with muscle weakness.
  • It is caused due to the deletion of the dystrophin gene.
  • Creatinine phosphokinase is usually elevated. Genetic testing will assist in diagnosing the patient. Muscle biopsy is also diagnostic.
71
Q

A 65-year-old male presented to a primary stroke center with a history of right-sided hemiparesis, which was greater in the legs than the arms and slurred speech that started earlier today. His speech was disinhibited. On examination, there was a sensory deficit in the perineum and right side of the leg. Which of the following is the most likely affected structure?

A

1. Left anterior cerebral artery
2. Right middle cerebral artery
3. Left middle cerebral artery
4. Right anterior cerebral artery

  • In this patient, the most likely diagnosis is anterior cerebral artery syndrome causing contralateral hemiparesis and sensory loss, which are more in lower extremities than the lower extremities and disconnection syndrome.
  • The major blood supply of the brain is by two sets of arteries, the vertebral arteries, and the carotid arteries. Inside the skull, the carotids divide into two major arteries, the anterior and the posterior cerebral arteries, and many smaller arteries like the ophthalmic artery and the anterior choroidal arteries. These arteries supply blood to the anterior two-thirds of the brain. The ophthalmic artery supplies the structures of the orbit while the posterior communicating artery acts as an anastomotic ‘connecting vessel’ in the Circle of Willis. Anterior choroidal artery supply structures in the brain important for motor control and vision, and the anterior cerebral artery supplies part of the cerebrum.
  • The vertebral arteries join to form a basilar artery which is located near the base of the skull. They give off many small branches in the brain stem and forms the posterior cerebellar and the posterior meningeal arteries. Meningeal branch supplies the falx cerebelli, a sheet of dura mater, while the anterior and posterior spinal arteries supply the spinal cord, spanning its entire length. The posterior inferior cerebellar artery supplies the cerebellum.
  • The anterior cerebral artery supplies the anteromedial portion of the cerebrum, whereas the middle cerebral artery is situated laterally, supplying the majority of the lateral part of the brain. The posterior cerebral artery supplies both the medial and lateral parts of the posterior cerebrum.
72
Q

A 24-year-old male is brought to the emergency department after a motorcycle collision. He was wearing a helmet but hit his head on the handlebars as he was thrown forward. His Glasgow Coma Scale score is 15, but he has difficulty smelling, which he believes is from clear ‘mucous’ draining from his nose. A fracture is noted on computed tomographic imaging of the skull. What is the most likely location of the injury that caused this patient’s symptoms?

A
  1. Nasal septal swell body
  2. Fovea ethmoidalis
  3. Crista galli of the ethmoid bone
    4. Cribriform plate of the ethmoid bone

  • The cribriform plate of the ethmoid bone forms the central portion of the roof of the nasal cavity, which separates the cranial vault from the external environment. On either side of the cribriform plate are the lateral lamellae, which extend vertically towards the foveae ethmoidalis, which in turn rise obliquely to join the roofs of the orbits.
  • The olfactory bulbs send projections through the cribriform plate to the olfactory mucosa at the apex of the nasal cavity. Because these nerves are very thin, they are prone to shearing injuries in the setting of blunt force head trauma. Shearing of these nerves will produce hyposmia or anosmia, depending on the severity of the injury.
  • Cerebral spinal fluid (CSF) is a clear watery liquid that main drain directly into the nasal cavity if the integrity of the anterior skull base is compromised. Testing the fluid for glucose, ß-2 transferrin, or ß-trace protein can identify CSF. Alternatively, if watery sanguineous rhinorrhea is present, placing a drop on a piece of filter paper and noting a halo sign may increase suspicion for a CSF leak.
  • An acute CSF leak is not a surgical emergency but may require lumbar drainage and bed rest in the short term and potentially surgical repair if conservative measures fail to resolve the leak within a week. The usefulness of antibiotic prophylaxis to prevent meningitis in these patients is controversial.
73
Q

A 45-year-old male presents with low back pain for the past three months and right leg weakness for the past one week. He also experiences numbness in the right foot. For the past two days, he has the feeling of incomplete evacuation of the bladder after voiding urine. On physical examination, he has grade 4+ power of right ankle dorsiflexion and sensory dullness along the lateral aspect of the leg. A magnetic resonance imaging scan of the lumbar spine is done, which shows a homogeneously enhancing tumor at the level of L2 extending proximally and involving the conus. There is a small T1-hyperintense region at the superior aspect of the lesion. He undergoes laminectomy and excision of the lesion. Which of the following immunophenotype markers is likely to be positive in this lesion?

A

1. CD99
2. CK5/6
3. CK20
4. E-cadherin

  • Astrocytoma and ependymoma are the most common intradural and intramedullary lesions of the spine. However, ependymoma (myxopapillary variant) is by far the most common lesion found in the conus medullaris.
  • The homogeneous enhancement and the sign of bleeding (T1 hyperintense region) suggest the diagnosis of ependymoma.
  • Myxopapillary ependymomas are usually positive for these immunophenotype markers: GFAP, S100, vimentin, CD99, AE1/AE3, and NCAM1.
  • Myxopapillary ependymomas are usually negative for these immunophenotype markers: CK5/6, CK20, E-cadherin, cytokeratin.
74
Q

A concerned father brings in his 2-year-old for evaluation of his low heart rate. While playing doctor with his toddler yesterday, he noted his child’s pulse to be low. Oddly, his toddler has stopped running as much and prefers to walk, holding an adult’s hands. If left to walk on his own, his gait is quite unsteady and looks almost as though he is learning to walk again. Of note, he started walking at 12 months of age. On examination, the child’s heart rate is 54 beats per minute. His temperature is 98.5 degrees Fahrenheit, respiratory rate is 35 beats per minute, and blood pressure is 110/65 mmHg. When the provider tests his gait, he seems unsteady and looks for something to hold onto to help him walk properly. On his ophthalmologic examination, his retinal vessel margins are blurry. The provider suspects an intracranial mass. Which of the following statements is true about the type of tumor most likely suspected?

A
  1. The peak incidence is at 20 years of age
    2. It typically arises from the vermis and the roof of the fourth ventricle
  2. Hydrocephalus is usually seen late in the course of the disease
  3. High tyrosine-protein kinase C receptors expression may be an indicator of poor prognosis

  • Medulloblastomas are invasive embryonal tumors of the cerebellum and tend to metastasize along the craniospinal axis.
  • Tumor growth starts along the floor of the fourth ventricle and occupies the entirety of the ventricle, subsequently invading both the cerebellar vermis and the brainstem, causing ataxia and signs of hydrocephalus early on.
  • The peak incidence of medulloblastoma is in the first decade of life, with an annual incidence of 0.5 per 100,000 children.
  • The mainstay of treatment for medulloblastoma is maximal surgical resection followed by chemotherapy and whole neuroaxis radiation.
75
Q

A 76-year-old woman was hit by a car while walking on the road. She is intoxicated with an open tibial fracture. The patient is immobilized on a longboard, and an extrication collar is used. The patient appears to have bent her upper thoracic spine forwards and is not properly touching the board. What is the next best step in the management of this patient?

A
  1. Remove the spine board
  2. Transfer the patient to another spine board
  3. Try to position the patient so that his upper thoracic region is touching the board
    4. Leave it as it is, provided the rest of the spine is in contact with the board, and support her with pillows

  • Hard collars and longboards can produce significant complications, and the quality of evidence in their use is very low. Increased intracranial pressure (even in normal subjects), skin breakdown, and an inability to access the airway are some issues with hard collars.
  • Clinical algorithms in determining spinal immobilization should be used.
  • The patient may be bending the spine due to spinal injury at that level.
  • The patient should not be forced to position such that the spine board will be in contact.
76
Q

A male newborn is delivered by assisted forceps delivery. He has torticollis immediately after birth. There is a preferential positional preference in the supine position while sleeping. He has abnormal flattening of the left posterior part of the head. No bony ridges are felt on palpation. Which of the following findings is most likely to be seen on a CT scan in this patient?

A
  1. Unilateral coronal synostosis on left
  2. Unilateral lambdoid synostosis on left
    3. A skull resembling a parallelogram, without synostosis
  3. Sagittal synostosis

  • The clinical scenario is in favor of deformational plagiocephaly (DP). In DP, there is no synostosis. The skull shape is that of a parallelogram.
  • Assisted labor and first-born male child are risk factors for DP.
  • Torticollis and preferred supine sleep position are also risk factors for DP.
  • In case of craniosynostosis, a palpable bony ridge can be felt.
77
Q

A 7-year-old girl presents with dizziness, blurry vision, and frequent falls in the past month. Her mother reports her daughter started experiencing headaches two weeks ago. Her vitals are normal, and the physical exam demonstrates gait instability and inability to abduct the left eye. No signs of systemic infection are present. A brain MRI with and without contrast is ordered, but a helical computed tomography scan shows signs of edema in the brain stem. What is the best next step in management?

A
  1. Await MRI brain results
    2. Intravenous corticosteroids
  2. Emergent radiotherapy
  3. Stereotactic biopsy

  • The most important goal of treatment is the stabilization of neurological symptoms.
  • Corticosteroids, especially dexamethasone, are the most common initially prescribed treatment for children with newly diagnosed (or suspected) diffuse intrinsic pontine glioma (DIPG).
  • Dexamethasone serves as a temporary measure for brain edema before undergoing stereotactic biopsy.
  • Surgery has a limited role in the management of DIPG due to its location. While the brain MRI may provide more detail, there is no need to delay administering steroids because the CT demonstrates brain edema. Radiotherapy is a mainstay of DIPG treatment and can lead to symptom relief, but administering steroids is the best next step in management.
78
Q

A 17-year-old male was brought to an emergency department after he was struck by a car. On arrival, his blood pressure was 112/74 mmHg, heart rate is 114/min, respiratory rate is 14 breaths/min, and oxygen saturation is 90% at room air. On physical examination, the patient had bruises and abrasion all over the body with a laceration of the abdomen. His Glasgow coma scale was 15/15. Examination of the left ear revealed purplish colored tympanic membrane. X-rays of upper and lower limbs demonstrated fractures of the left radius and the left femur. CT scan of the abdomen showed irregular linear areas of hypoattenuation. The patient was immediately shifted to the operating room. He had several surgeries and was improving but still required morphine for pain control. One week after the surgery, he develops a fever and altered mental status. His Glasgow coma scale becomes 8/15. What is the next best step in the management of this patient?

A

1. CT of head and lumbar puncture to rule out meningitis
2. Order a CT scan to rule out a pulmonary embolism
3. Check blood cultures and start antibiotics for sepsis
4. Chest x-ray to evaluate for atelectasis secondary to hypoventilation

  • A basilar skull fracture allows communication between the nasopharynx or middle ear and the meningeal space. Fractures that involve the petrous ridge of the temporal bone will cause blood to pool behind the tympanic membrane causing it to appear purple. This usually appears within hours of injury and maybe the earliest clinical finding.
  • Altered mental status and fever with known basilar skull fracture require evaluation of the CSF. CSF leak is not easy to diagnose and the fluid should be sent for analysis of beta transferrin.
  • The majority of CSF leaks resolve spontaneously within 5-10 days but some can persist for months. Meningitis may occur in less than 5% of patients but the risk increases with the duration of the CSF leak.
  • Pulmonary emboli are rare in children but a fat embolus could occur. Hypoventilation and atelectasis would have more respiratory symptoms.
79
Q

A 32-year-old patient with post-traumatic diffuse brain edema was being monitored clinically by the placement of the intraventricular intracranial pressure (ICP) monitoring system. Suddenly his attending nurse sees a gradual rise in the P2 wave going above the P1 and P3 waves, as visualized in the ICP waveform analysis. There is also a fall in his motor score from M5 (localizing) to M3 (flexing.) Which of the following is the appropriate immediate next step in managing the patient?

A
  1. Prepare for decompressive hemicraniectomy
  2. Take the patient for an emergency repeat CT scan
  3. Paralyze and sedate the patient
    4. Lower down the external ventricular drain (EVD) below the level of the tragus

  • The clinical scenario, as depicted in the ICP waveform analysis, is typical of rising ICP pressure; therefore, there is an urgent need for intervention to reduce the same.
  • The intraventricular ICP monitoring system, in addition to being the gold standard for measuring the same, allows for the immediate release of CSF, thereby counteracting any rise in ICP.
  • So the best immediate management plan would be to allow for egress of CSF by lowering the transducer and the EVD bag below the level of the tragus (external auditory canal).
  • After combating this sudden rise in ICP only, we can ask for other interventions like repeating CT scans and preparing for damage control surgery.
80
Q

A 16-year-old boy presents to the clinic with features suggestive of palmer hyperhidrosis. It has caused significant psychological distress to the patient, owing to its interference with his social interactions. The treating clinician counsels him of the probable role of stellate ganglion block in managing the condition. The patient is anxious about the probable complications of the interventional procedure. Which of the following methods is least likely to cause postprocedure complications in this patient?

A
  1. Blind manual approach
    2. Ultrasound guidance
  2. Fluoroscopic guidance
  3. CT guidance

  • Pre-procedural ultrasound helps in assessing the relevant anatomic study and relationship between the carotid sheath, esophagus, longus colli muscle, tubercle of the transverse of the C6, and the vertebral artery. This helps in assessing the safe trajectory of the injection during the procedure.
  • The ultrasound also provides continuous and real-time images of the trajectory of the tip of the needle, thereby preventing any vascular injuries and inadvertent damages to the esophagus and thyroid gland.
  • It also helps in determining the precise subperiosteal location of the tip of the needle while injecting the drug and also of the pattern of the distribution of the drug, to ensure its maximum efficacy. Moreover, it has a low learning curve and avoids the risk of radiation exposure.
  • The blind manual approach has the risk of causing injury to the vertebral artery and damage to the pleura, causing a pneumothorax. The efficacy of the procedure is also low compared to that of the ultrasound-guided procedure. Fluoroscopic guidance has the risk of radiation exposure. It also cannot predict inadvertent vascular injuries.
81
Q

A 55-year-old man presents to the clinic for evaluation. The patient has a complicated medical history with asthma, diabetes, hypothyroidism, and peripheral neuropathy. He
has a BMI of 36 kg/m2. Upon further history and examination, he reports a long-standing history of back pain since his mid-20s associated with his job as a construction worker. Since then, he has tried physical therapy, chiropractic, aquatics, spinal injections, and opioid therapy. He had previous radicular symptoms going down into the right leg. However, these resolved after six months of continuous gabapentin use. His exam reveals diminished lumbar spinal flexion and extension with no other signs. He has normal reflexes with no focal motor weakness. His MRI reveals Modic type vertebral endplate changes without canal stenosis or evidence of facet joint arthropathy. Which of the following is the next best step in the management of this patient?

A
  1. Transforaminal epidural corticosteroid injection
  2. Lumbar medial branch block
  3. Vertebroplasty
    4. Basivertebral nerve neurotomy

  • Basivertebral nerve ablation/neurotomy would be clinially indicated in this scenario. The procedure is indicated when patients meet the following indication criteria: chronic axial low back pain (greater than 6 months of duration); pain refractory to conservative nonsurgical treatment for at least six months of duration; and evidence of vertebral endplate changes on diagnostic images (MRI).
  • MRI findings compatible with vertebral endplate changes indicated in basivertebral nerve ablation include Modic type I and/or Modic type II changes; vertebral endplate changes with inflammation, edema, disruption, and/or fissuring; fibrovascular bone marrow changes (hypointensive signal for Modic type I changes); fatty bone marrow replacement (hyperintensive signal for Modic type II changes).
  • Basivertebral nerve ablation/neurotomy is the only intervention to directly target chronic axial low back pain of vertebral nature when other anatomical structures are not involved, such as facet joints, intervertebral discs, central canal, etc.
  • Lumbar epidural corticosteroid injection is clinically indicated for lumbar radiculopathy. Lumbar medial branch block is indicated for facet arthropathy, and vertebroplasty is clinically indicated in vertebral compression fractures.
82
Q

An 8-month-old girl child is brought to the outpatient department with a history of reduced activity and headaches over the past two days. These are associated with vomiting. On examination, the child is drowsy and anterior fontanelle is bulging but has no neurological deficits. She undergoes a magnetic resonance imaging of the brain, which shows a T1- hypointense and T2-hyperintense lesion of size 2 cm in the tectum of the midbrain, which is not enhancing on contrast and is causing obstructive hydrocephalus. She undergoes an emergency ventriculoperitoneal shunt on the right side. Following surgery, she recovers completely and is discharged after three days. Six weeks later, she presents to the outpatient department for follow-up. Which of the following will suggest whether the shunt is working properly?

A
  1. Postoperative magnetic resonance imaging showing smaller tumor size
  2. Increased head circumference
    3. Smaller suture line
  3. Large anterior fontanelle

  • A smaller suture line correlates with proper shunt function in children with open sutures.
  • Signs of obstruction include enlargement of the infant’s head, fontanelle full and tense when the infant is upright and quiet, prominent scalp veins, swelling along the shunt tract, vomiting, sleepiness, irritability, downward deviation of the eyes, poor feeding.
  • When an abrupt malfunction of a shunt occurs, symptoms can develop very rapidly and lead to coma or death.
  • Ventricular shunting for obstructive hydrocephalus secondary to a tumoral mass decreases ventricle size but does not decrease tumor size.
83
Q

While investigating his severe and sudden-onset headache, a 65-year-old patient was found to have a ruptured anterior communicating artery aneurysm. The relatives are worried about the possible risk of vasospasm following the clipping of the aneurysm. What intraoperative modality is ideal in helping the surgeon minimize the risk and opting for early therapeutic intervention in managing the same?

A
  1. Endoscopy
  2. Angiogram
  3. Indocyanine green (ICG) study
    4. Flow 800 study

  • There is a risk of vasospasm in the postoperative period in patients undergoing surgery following subarachnoid hemorrhages from aneurysmal rupture. The release of bilirubin for the lysed blood and inadvertent clipping of the perforators and parent vessels predisposes to vasospasm.
  • Flow 800 is an automated software that displays flow mapping generated from the indocyanine green angiography of the parent vessels, their branches, and the perforators. It also analyzes the pattern of a flow velocity as well as transit time through them.
  • The difference in flow velocity of more than 50% between the parent and the branching vessels and the significant increment in the transit time through them are factors that predict a high risk for postoperative vasospasm.
  • Intraoperative angiography is technically demanding, along with the risk of radiation exposure. Indocyanine green only gives a qualitative analysis of flow patterns through the vessels. Endoscopy helps in evaluating the blind corners of the operating fields but will not give any information with regard to patterns of flow dynamics.
84
Q

A 30-year-old man presents to the outpatient department with complaints of blurring of his left eye vision, a subjective pulsatile orbital bruit, mild protrusion of the same eye, and headache over the past two months. He reveals that he had been treated for a head injury five months back. On examination, his visual acuity is 20/70 in the left eye, and there is mild proptosis. He undergoes a computed tomogram angiography of the brain, which shows an enlarged superior ophthalmic vein and a bulging cavernous sinus on the left side with abnormal enhancement of the left cavernous sinus. Subsequently, he undergoes a digital subtraction angiogram, which shows a meningeal branch of the left external carotid, forming a fistulous connection with the left cavernous sinus. He is being planned for endovascular treatment of this condition. Which is the preferred route?

A
  1. Transarterial embolization via the ipsilateral internal carotid artery
  2. Transvenous embolization of the cavernous sinus via the superior ophthalmic vein through transfacial catheterization
    3. Transvenous embolization via the ipsilateral inferior petrosal sinus
  3. Transvenous embolization via the superior ophthalmic vein through direct surgical cannulation

  • Transvenous embolization via the inferior petrosal sinus is the preferred treatment modality for indirect carotid-cavernous fistulas (Barrow type B, C, D). The inferior petrosal sinus may be accessed via the jugular vein.
  • The superior ophthalmic vein is a potential route to the cavernous sinus, most commonly employed in scenarios in which the inferior petrosal sinus is stenosed or in discontinuity with the jugular system. It may be accessed endovascularly through the facial vein, the preferred method.
  • Direct surgical cannulation of the superior ophthalmic vein is typically reserved for cases in which other transvenous approaches to the cavernous sinus have been exhausted, and transarterial access is not appropriate or sufficient. Endovascular access via the facial vein is preferred but may be technically challenging.
  • Transarterial embolization via the internal carotid artery is appropriate for direct carotid-cavernous fistulas (Barrow type A). Indirect and mixed-type fistulas require at least partial venous access.
85
Q

A 16-year-old girl is being evaluated for recent- onset seizures. CT head reveals a frontal lobe lesion with areas of stippled calcification. Histology of the lesion following surgical resection shows cellular monotony, uniform cell density, branching capillaries, and nuclei surrounded by a rim of clear cytoplasm. Which of the following factors is most likely to influence the prognosis in this patient?

A
  1. Age at diagnosis
  2. Seizure free period
  3. Absence of metastasis
    4. Isocitrate dehydrogenase 1 and 1p19q co-deletion

  • Infiltration along with moderate cellularity and branching capillaries that give a chicken-wire appearance are the main microscopic feature of oligodendroglioma. The presence of vascular proliferation or necrosis should raise concern for higher tumor grade such as anaplastic oligodendroglioma.
  • The typical histologic appearance of oligodendroglioma demonstrates nuclei with a rim of clear cytoplasm, resulting in the classic fried egg appearance due to artifactual cytoplasmic retraction that happens as a result of the fixation process.
  • Isocitrate dehydrogenase (IDH) mutation with 1p19q codeletion is the defining genetic marker for oligodendroglioma. These two mutations are predictive of a better therapeutic response to chemotherapy (temozolomide) and combined chemoradiotherapy, with longer overall survival compared to patients with other types of glioma without these genetic changes.
  • The tumor histology, grade, size, and anatomic structures involved are all very important considerations to determine patient prognosis because complete tumor resection is the goal for optimal treatment. If the tumor cannot be completely resected, the tumor can continue to be a seizure focus and may later undergo degeneration to more malignant glioma. Metastasis is very rare in oligodendroglioma, but when present portends a poor prognosis.
86
Q

A 60-year-old man presents with a headache following a motor vehicle accident four hours ago. He was the unrestrained passenger in a rear-end automobile accident. He did not hit his head during the accident but says that his neck sustained a rapid back-and-forth movement that caused him some discomfort after the accident. Otherwise, he felt fine. One hour later, he developed a severe right-sided headache over the right frontal and temporal regions. He also complained of transient blurry vision that resolved after thirty minutes. He has never had symptoms like this before. His past medical history is significant for hypertension, for which he takes hydrochlorothiazide. The rest of his past medical history is unremarkable. He smokes half a pack of cigarettes a day and does not consume alcohol. His family history is significant for cluster headaches in his father and brother and migraine headaches in his mother. On examination, blood pressure is 150/89 mmHg, pulse is 90 beats/min, and respiratory rate is 19 breaths/min. The patient is sitting down in apparent distress, holding his right head. His right pupil is dilated compared to the left pupil. There is no ptosis or anhidrosis. Laboratory studies show hemoglobin of 13 g/dL, platelets of 250,000/mm3, and WBCs of 6,000/mm3. An X-ray of the neck and spine shows no abnormalities, and a noncontrast CT scan of the head shows no hemorrhage or mass lesions. Which of the following is the best next step in the management of this patient?

A
  1. Inhalation of 100% oxygen
    2. Head and neck CT scan with contrast
  2. Intravenous naproxen and a follow-up visit after four weeks
  3. Echocardiogram

  • This patient is presenting with unilateral headache and same- sided partial Horner syndrome after a motor vehicle accident resulting in a whiplash neck injury (a forceful, rapid, back-and- forth movement of his neck).
  • Unilateral headache and same sided partial Horner syndrome (no ptosis or anhidrosis) indicated internal carotid artery dissection until proven otherwise.
  • Internal carotid artery dissection can be caused by trauma (e.g., neck whiplash injury) or can occur spontaneously. It leads to the formation of an intramural hematoma that can obstruct blood flow and compress the sympathetic nerves that travel alongside the internal carotid artery. Anhidrosis is typically absent because the sweat fibers branch of the superior cervical ganglion and travel alongside the external carotid artery.
  • The diagnosis is made by a CT or MR angiography. Treatment is similar to the treatment of a stroke, entailing thrombolysis within four and a half hours of symptom onset and antiplatelet therapy. Cluster headache has a very similar presentation, but the headache is usually short-lived (30 mins-2hrs). Given this constellation of symptoms occurring after a neck whiplash injury, ruling out carotid artery dissection is the best next step in the management of this patient.
87
Q

A male infant is delivered at 29 weeks gestation. A brain MRI reveals the presence of a cavity of the midline with a triangular shape with a wide base dorsally and the apex pointing anteriorly reaching as far forward as the foramen of Monroe. Which of the following is the most likely longterm outcome for this condition?

A
  1. Transient increased intracranial pressure
    2. No clinical sequelae
  2. Hydrocephalus
  3. Psychosis spectrum disorders

  • Cavum veli interpositi (CVI) is usually an incidental finding at an MRI or CT scan mainly asymptomatic.
  • Transient increased intracranial pressure related to CVI is a rare occurrence. It is related to the ball-valve mechanism between the cisterna CVI and the cisterna venae magnae Galeni with transient increased intracranial pressure.
  • Hydrocephalus related to CVI is a rare occurrence. It is also related to the ball-valve mechanism between the cisterna CVI and the cisterna venae magnae Galeni with a severe increase of the intracranial pressure.
  • The presence of the CVI in children has been related to the development of psychosis spectrum disorders; however, further studies are needed to confirm this association.
88
Q

A 67-year-old woman presents to the clinic with right-sided facial pain. It has been present for 4 years, is located on the right cheek, and it feels like intermittent electric shocks. Chewing can exacerbate these symptoms. She has been prescribed carbamazepine in the past with some success, but she no longer can tolerate this medication due to side effects. She is seeking a permanent procedure to help with her pain. Radiofrequency ablation of the implicated ganglion is planned. Which of the following vessels is most likely to be injured as a result of this procedure?

A
  1. External carotid artery
  2. Internal carotid artery
    3. Maxillary artery
  3. Superficial temporal artery

  • This patient has trigeminal neuralgia and could benefit from a sphenopalatine ganglion radiofrequency ablation.
  • The needle entry point is nearest to the maxillary artery, which is a branch of the external carotid artery.
  • The maxillary artery is the most likely vessel to be injured with a sphenopalatine ganglion radiofrequency ablation procedure due to its anatomic location being near the site of needle entry, which is anterior to the mandible and inferior to the zygomatic arch.
  • The internal carotid artery is less likely to be injured compared to the maxillary artery when performing a sphenopalatine ganglion radiofrequency ablation. The superficial temporal artery is potentially in the territory of the needle entry point for a sphenopalatine ganglion radiofrequency ablation; however, the maxillary artery is closer to the needle entry point and is more likely to be injured.
89
Q

A 29-year-old man is brought to the hospital after a bar fight. He is found to have a minimally displaced zygomatic fracture. The orbital floor is involved on the CT scan, and there appears to be blood in the maxillary sinus as well. The decision is made to pursue nonoperative management, and the patient is discharged home. Which of the following is the next best step in the management of this patient?

A
  1. Penicillin
  2. Co-amoxiclav
  3. Metronidazole
    4. Symptomatic care only

  • The existing literature does not support routine prophylactic antibiotic use in nonoperative facial fractures.
  • Most surgeons use perioperative antibiotic therapy (defined as 2 hours before surgery to 24 hours post-surgery) when managing facial fractures.
  • There may be a role for postoperative prophylactic antibiotic use, but this is very controversial.
  • If one chooses to prescribe antibiotics, an antibiotic that covers sinonasal flora should be selected.
90
Q

A 45-year-old man presents to the clinic for evaluation of lower back pain. He has had this pain for several months now and would like an intervention. His vitals signs are unremarkable. His physical exam demonstrates only back pain and is otherwise non-contributory. His lab results are all within normal limits. Iontophoresis is being considered. Which of the following physiological features in this patient would allow for the most efficacious analgesia?

A

1. Higher pore density
2. Thicker stratum corneum
3. Increased pigmentation
4. Cold temperature

  • Iontophoresis is affected by individual patient physiology.
  • Pore size, blood flow, temperature, and pigmentation can all affect the permeation of iontophoretic meds.
  • Pores offer routes of least resistance to transdermal delivery.
  • Medications are more likely to permeate in patients whose skins have higher pore densities.
91
Q

A 65-year-old male patient presents with progressive, nonfluctuating, right-sided hearing loss and tinnitus, and unsteadiness for the past two months. He has no known medical comorbidities. On examination, vital signs are within normal limits. He is awake, alert, and oriented to 3 spheres. Pupils are isochoric, equally reactive to light with full extraocular muscle movement. There are decreased forehead creases and a shallow nasolabial fold on the right. Weber test lateralizes to the left, and bone conduction (BC) is better than air conduction (AC) on the right. The rest of the cranial nerve examination is normal. The motor and sensory examinations are normal. He is able to ambulate without assistance but veers to his right side. When asked to do tandem gait, he falls on the right side. Reflexes are normal, and Babinski is negative bilaterally. A cranial MRI with contrast is requested. What is the most expected finding in this patient?

A
  1. A fluid signal in the middle ear cavity and mastoid antrum
    2. A well-circumscribed enhancing lesion at the cerebellopontine angle
  2. T2 lesions consistent with MS
  3. A well-circumscribed enhancing lesion on the cerebellum

  • An acoustic neuroma is a benign tumor that may be locally destructive. It typically produces a unilateral, progressive, nonfluctuating, neurosensory hearing loss with unilateral tinnitus and unsteadiness.
  • The diagnosis of an acoustic neuroma is made with a contrast magnetic resonance imaging (MRI) or a computed tomogram scan. Contrast is essential; otherwise, the non-enhanced scan can miss small tumors.
  • These tumors usually occur adjacent to the cochlear or vestibular nerve. Anatomically, acoustic neuromas tend to occupy the cerebellopontine angle.
  • The majority of acoustic neuromas present with unilateral hearing loss due to cochlear nerve interruption or impairment of blood supply to the nerve. Other clinical features include tinnitus, decreased word understanding, vertigo, headaches, and facial numbness. With enough growth, the mass of the cerebellopontine angle will eventually compress the brainstem and cause gait abnormalities.
92
Q

A 60-year-old man undergoes intrathecal catheter implantation (T12-L1) for morphine infusion to manage intractable low back pain post failed back surgery. In the immediate postoperative period, the patient has a good response, with a notable reduction in pain from 8/10 to 2/10. After a few weeks, the patient experiences new onset lumbar pain different from his initial pain. He describes it as a radicular pain radiating to the buttocks, groin, and back of the left lower limb. Which of the following is most likely to cause these symptoms?

A

1. Catheter migration and twisting around a nerve root
2. Opioid toxicity
3. Catheter occlusion or pump malfunction
4. Granuloma of the catheter tip

  • A patient with an intrathecal catheter showing new-onset neurological symptoms has a high percentage of suspected catheter migration. First of all, imaging of the spinal axis is necessary to exclude stenosis or the formation of a granuloma. Intrathecal catheter-tip granulomas are rare and are formed progressively over time.
  • Neuroimaging allows visualization of the catheter and possible compression of nerve roots.
  • Although the worsening of the painful manifestations may be suggestive of catheter occlusion or catheter twisting, the appearance of new neurological symptoms (pain radiating to the buttocks, groin, and back of the lower limbs) can be better explained by the compression of nerve roots.
  • Opioid toxicity occurs mainly as respiratory depression and somnolence.
93
Q

A 48-year-old man presents to the clinic with clumsiness of the right hand and difficulty in walking. The physical examination reveals a positive Hoffmann sign and guttering of intermetatarsal space. MRI shows cervical spondylosis and spinal canal stenosis. Lateral flexion radiograph reveals cervical kyphosis of more than 30 degrees. What is the most appropriate management strategy for this patient?

A
  1. Decompression with posterior fusion
    2. Decompression with anterior fusion
  2. Decompression and laminectomy
  3. Physical therapy

  • The above-mentioned patient has significant cervical spondylosis with kyphosis.
  • The appropriate treatment option is spinal canal decompression and anterior cervical fusion.
  • In kyphosis, anterior fusion is recommended.
  • Decompression and laminectomy without fusion lead to poor results.
94
Q

A hospital staff member who performs work in an x-ray laboratory for interventional procedures while standing close to the x-ray tube should wear what protective device to record and audit their radiation exposure levels?

A

1. A dosimeter under and over the apron and optional dosimeters in various locations above the apron to estimate doses in potentially- exposed areas
2. Protective material only, and no dosimeter, because doses are negligible when protection is used
3. Protective devices only, and no dosimeter, because doses should be monitored via environmental monitoring methods
4. Dosimeter at the thorax level only with no shielding material or apron, so that the dosimeter has an exact indication of the actual risk incurred

  • Dosimeters are devices that measure the absorbed dose of radiation that contacts the caretaker. These devices should be worn by all hospital staff who encounter planned ionizing radiation. This allows the radiation safety department to audit exposure for individual staff and inform them of their occupational exposure levels. Staff should be aware of their dose levels and the dose limits set forth by their institution.
  • Dosimeters should be worn both outside and inside the leaded apron for comparison of doses, and the dosing should be analyzed by the facility’s radiation safety department.
  • Raising awareness of the importance of dosimetry should be a priority for the occupational safety or radiation safety departments in health systems.
  • Sanchez et al. reported that as much as 50% of physicians do not wear or incorrectly wear dosimeters. Unfortunately, in a significant number of healthcare settings, there is a paucity of monitoring and, thus, a lack of reliable data.
95
Q

Which of the following is the most important clinical significance of the lambda waves?

A
  1. Saccadic visual exploration
  2. Coma
  3. Persistent vegetative state
  4. Seizure

  • The clinical significance of LWs could be to demonstrate the physiological functioning of the saccadic visual exploration processes. In other words, as LWs are probably related to an oculomotor visual integration mechanism.
  • Lambda waves are physiological, sharp, and triangular transients seen over the occipital regions when the eyes are open.
  • Lambda waves typically present with saccadic eye movements during visual scanning.
  • They disappear when the eyes are closed and during sleep.
96
Q

A 65-year-old female presents to the primary care clinic for follow-up after having been in a car accident three months ago. She remembers her constant kidney problem was what caused her to feel dizzy and crash her car. She seems worried and describes having had multiple episodes of loss of consciousness and sensations she describes as “weird” with shaking of her body. She has tried NSAIDs, ice baths, increasing her hydration, and sleeping more to try to relieve these symptoms with no avail. On physical exam, she appears alert and oriented to person, time, and place. Mini-mental status exam is normal. She asks if there is any pharmaceutical management that could help stop these episodes. What agent would be a suitable treatment for her episodes?

A
  1. Valproate
  2. Ethosuximide
  3. Phenobarbital
    4. Phenytoin

  • Phenytoin is a commonly used anticonvulsant and rarely causes nephrotoxicity.
  • Phenytoin is associated with gingival hypertrophy.
  • Phenytoin can induce the P-450 enzymes in the liver. Thus, reducing the bioavailability of other drugs.
  • Ethosuximide is primary for the absence of seizures occurring in children.
97
Q

A 16-year-old male presented to his primary care provider with right earache and pus discharge for the past four days. He had flu one week ago after which his current symptoms developed. He was started on oral antibiotic therapy. After one week, he presents to the emergency department with neck rigidity and projectile vomiting for the past 2 hours. Computed tomography (CT) of the head reveals partial opacification of the mastoid air cells and leptomeningeal enhancement. What is the most likely diagnosis of this patient?

A
  1. Sinusitis
  2. Postauricular abscess
    3. Mastoiditis
  3. Acute otitis media

  • The patient initially had an upper respiratory tract infection, which led to Eustachian tube dysfunction. Decreased clearance of bacteria led to acute otitis media.
  • If antibiotic therapy is not effective, intratemporal, or intracranial complications can develop. This patient has developed mastoiditis, which is an intratemporal complication of acute otitis media.
  • Opacification of mastoid air cells, as seen on CT of the head, is suggestive of mastoiditis. The infection spreads into the meninges, which further causes meningitis.
  • When the diagnosis of otitis media is established, the goal of treatment is to control pain and to target the infectious process with antibiotics.
98
Q

A 17-year-old girl presents with a three-month history of recurrent lower back pain. The patient is a semi- professional dancer and says the pain starts soon after she finishes practice. It then improves over one to two hours. She says she would like to increase the training regime but is currently unable to because of the pain. She has a past medical history of recurrent shoulder dislocation. Which of the following findings is most likely expected in this patient’s condition?

A

1. Pain with lumbar spine hyperextension
2. Numbness over the lateral aspect of the foot
3. Achilles tendon tightness
4. Waddle gait

  • This young gymnast with a potential diagnosis of hypermobile joints presents with back pain associated with training. The history is typical for lumbosacral spondylolisthesis.
  • In highly functioning, the physical examination of patients with low-moderate grades of spondylolisthesis is unlikely to reveal any significant findings. Lumbar spine hyperextension is often the only positive provocative sign present on examination.
  • Pain on back hyperextension is the most common finding followed by hamstring contractures and, in severe cases, L5 nerve root signs.
  • Although in high-grade lumbosacral spondylolisthesis, the L5 nerve root can be affected, the numbness would be expected in the first web space, not over the lateral aspect of the foot. Hamstring tightness is more often associated with lumbosacral spondylolisthesis. Waddle gait can be associated with high- grade cases with significant compression of the L5 nerve root. In this scenario, the patient can practice gymnastics and is limited mainly by activity-related back pain. She would be unlikely to perform with advanced L5 nerve root weakness.
99
Q

A 65-year-old male underwent surgery for a lesion at the cerebellopontine angle two days ago. The patient presents to the emergency department with complaints of headache, meningismus, and mild confusion. The pathologist reported that the lesion is compatible with an epidermoid tumor. A lumbar puncture is performed, and the sample is sent for a cerebrospinal fluid routine examination. No bacteria are present on gram staining, and there is an increased white blood cell count. What is the most probable diagnosis?

A
  1. Subarachnoid hemorrhage
  2. Bacterial meningitis
    3. Aseptic meningitis
  3. Post-surgical meningitis

  • During resection of an epidermoid tumor, keratin may leak into the surrounding tissue and cause aseptic meningitis.
  • Aseptic meningitis symptoms after resection of an epidermoid tumor can be reduced with the use of corticosteroids during the postoperative period.
  • Removal of keratin debris decreases the incidence of aseptic meningitis.
  • Bacterial meningitis is rare on the second postoperative day and will usually have a pathogen identified on gram staining.
100
Q

A 58-year-old security guard presents after being hit on the side of his head with a baseball bat during an altercation. The examination does not demonstrate muscle weakness, and a CT scan of the head does not reveal any obvious facial nerve injury. He does have some clear rhinorrhea, however. Three days following the altercation, he develops ipsilateral complete facial weakness with drooling. What is the most appropriate treatment at this time?

A
  1. Transmastoid facial nerve decompression
  2. Intramuscular ceftriaxone
    3. Oral prednisone
  3. Translabyrinthine facial nerve decompression

  • This patient has delayed-onset facial nerve palsy, which is common in temporal bone fractures.
  • Delayed facial nerve palsy occurs in approximately 76% of patients who experience traumatic facial nerve palsy.
  • These patients generally have a good prognosis. One study found that 100% of their 820 patients with delayed onset facial weakness returned to House–Brackmann grade 2 function or better.
  • High-dose steroids are the treatment of choice for acute facial palsy. CSF rhinorrhea suggests trauma to the skull base and may indicate the need for prophylactic antibiotics, although this is somewhat controversial.